1 Introduction

In the study of liquid crystals one often encounters elastic energies with anisotropy, i.e. energies with distinct coefficients multiplying the square of the divergence and the curl of the order parameters. Typical examples involve the Oseen–Frank model [30], Ericksen’s model [43, 44] and the Landau–De Gennes model [5]. From a microscopic point of view, the anisotropy of these models can be interpreted as excluded volume potential of molecular interaction, cf. [29]. Anisotropic models also arise in the theory of superconductivity, cf. [10]. The anisotropy brings various new challenges to the studies of both variational problems and their gradient flows of the aforementioned models. In contrast to the convergence analysis of isotropic models, i.e. the (scalar) Allen–Cahn equations (cf. [6, 19, 34, 47, 48, 50, 53, 54]), the powerful analytic tools such as maximum principle and monotonicity formula are not readily established for anisotropic ones.

The attempt of this work is to study an anisotropic system modeling the isotropic-nematic phase transition of a liquid crystal droplet. Let \(d\in \{2,3\}\) be the dimension of the physical domain \(\Omega \) with \(C^3\) boundary \(\partial \Omega \). We consider the anisotropic Ginzburg–Landau type energy

$$\begin{aligned} A_\varepsilon ({\textbf{u}})=\int _{\Omega } \left( \frac{\varepsilon }{2} \mu |{\text {div}}{\textbf{u}}|^2+\frac{\varepsilon }{2} |\nabla {\textbf{u}}|^2+\frac{1}{\varepsilon }F({\textbf{u}}) \right) \, dx. \end{aligned}$$
(1.1)

Here \({\textbf{u}}=(u_1,u_2,u_3):\Omega \subset {\mathbb {R}}^d\mapsto {{\mathbb {R}}^3}\) is the order parameter describing the state of the system. The function \(F({\textbf{u}})\) is a double equal-well potential which permits the isotropic-nematic phase transition. More precisely, it attains its global minimum value 0 at \(\{ 0\}\cup {\mathbb {S}}^2\). An example of F is the Chern–Simons–Higgs model \(F({\textbf{u}})= |{\textbf{u}}|^2(1-|{\textbf{u}}|^2)^2\). See for instance [31, 36] for the physics and [9, 27, 28] for the mathematical analysis of related variational problems. The parameter \(\varepsilon >0 \) denotes the relative intensity of elastic and bulk energy, which is usually quite small. The parameter \(\mu >0\) is material dependent which measures the degree of anisotropy.

The energy (1.1) is a simplified case of the full Landau–De Gennes energy (cf. [35, 45]). The variational investigations of the isotropic-nematic phase transition involving (1.1) were first done by Golovaty, Novack, Sternberg and Venkatraman [27, 28] in the static case in 2D. The present paper is concerned with the \(L^2\)-gradient flow of (1.1), i.e. the following system.

$$\begin{aligned} \partial _t {\textbf{u}}_\varepsilon -\mu \nabla ({\text {div}}{\textbf{u}}_\varepsilon )&= \Delta {\textbf{u}}_\varepsilon - \frac{1}{\varepsilon ^2} D F ({\textbf{u}}_\varepsilon ){} & {} ~\text {in}~ \Omega \times (0,T), \end{aligned}$$
(1.2a)
$$\begin{aligned} {\textbf{u}}_\varepsilon (x,0)&={\textbf{u}}_\varepsilon ^{in}(x){} & {} ~\text {in}~\Omega , \end{aligned}$$
(1.2b)
$$\begin{aligned} {\textbf{u}}_\varepsilon (x,t)&= 0{} & {} ~\text {on }\partial \Omega \times (0,T), \end{aligned}$$
(1.2c)

where \( D F ({\textbf{u}})\) is the gradient of \(F({\textbf{u}})\) with respect to \({\textbf{u}}\). We shall study the small \(\varepsilon \)-asymptotics of this system with well-prepared initial datum \({\textbf{u}}_\varepsilon ^{in}\) that undergoes a sharp transition across a co-dimensional one interface \(I_0\subset {\mathbb {R}}^d\). We shall show that the energy density \(\frac{\varepsilon }{2} |\nabla {\textbf{u}}_\varepsilon |^2+\frac{1}{\varepsilon }F({\textbf{u}}_\varepsilon )\) will be concentrated on a mean curvature flow \(I:=\bigcup _{t\geqslant 0} I_t\times \{t\}\) starting from \(I_0\), namely

$$\begin{aligned} \lim _{\varepsilon \rightarrow 0} \int _{\Omega } \left( \frac{\varepsilon }{2} |\nabla {\textbf{u}}_\varepsilon |^2+\frac{1}{\varepsilon }F({\textbf{u}}_\varepsilon ) \right) \, dx=\sigma {\mathcal {H}}^{d-1}( I_t), \end{aligned}$$
(1.3)

where \({\mathcal {H}}^{d-1}\) is the \((d-1)\) dimensional Hausdorff measure, and \(\sigma \) is a positive constant depending on F. Moreover, we shall derive bulk limit \({\textbf{u}}:=\lim _{\varepsilon \rightarrow 0}{\textbf{u}}_\varepsilon \) away from \(I_t\) and its boundary condition on \(I_t\).

Fig. 1
figure 1

\(I_t\) is the interface, \(\Omega _t^+\) is the nematic phase and \(\Omega _t^-\) is the isotropic phase

System (1.2a) is a vectorial and anisotropic generalization of the scalar parabolic Allen–Cahn equation. In the scalar case, there have been many developments on its co-dimensional one limit to the (two-phase) mean curvature flow during the last two decades. Here we mention two classes of results and postpone the discussions of some others in the sequel. One is the convergence to a Brakke’s flow by Ilmanen [34] using a version of Huisken’s monotonicity formula [32] and tools from geometric measure theory. See also [11, 33, 47, 48, 50, 54] and the references therein for further renovations. Despite of its energetic nature, a major difficulty of such an approach is the control of the so-called discrepancy measure, and almost all existing literatures using this approach rely crucially on a version of Modica’s maximum principle [46]. However, it is not clear whether Modica’s maximum principle holds for elliptic/parabolic systems. Another approach, which relies more on parabolic comparison principle, is the global in time convergence towards the viscosity solution built by Chen–Giga–Goto [13] and independently by Evans–Spruck [20]. Such an approach has been implemented by Evans–Soner–Souganidis [19]. One can also refer to [6, 53] and the references therein for further discussions. These two approaches both give global in time (weak) convergences to weakly defined solutions of the mean curvature flow (up to their extinction times). However, as their technics involve parabolic maximum principle in one way or another, it is not clear how to use them to attack vectorial models in general. It is worth mentioning that for radially symmetric initial datum, Bronsard–Stoth [8] have obtained global in time convergence to the mean curvature flow of planar circles.

To the best of our knowledge, there are mainly two approaches to rigorously justify the convergence of the vectorial Allen–Cahn equations, both assuming that the limiting interface motion has a (local in time) classical solution. Compared with the aforementioned methods, which lead to global in time (weak) convergence, they have quite different natures. The first approach is the asymptotic expansion technics developed by De Mottoni–Schatzman [15] and by Alikakos–Bates–Chen [1]. It has been used recently by Fei–Wang–Zhang–Zhang [22] to study the isotropic-nematic phase transition in liquid crystals, and by Fei–Lin–Wang–Zhang [21] to study matrix-valued Allen–Cahn equations.

The second approach, which also assumes a classical solution of the limiting interface motion (but not the limiting flows in the bulk regions), is the modulated energy method developed by Fischer–Laux–Simon [24]. Such a method is motivated by Jerrard–Smets [37] and Fischer–Hensel [23], and has been generalized to a matrix–valued model by Laux–Liu [40].

In the present work, we shall use the methods employed in [24, 40] to derive the energy convergence (1.3) and the bulk limit \({\textbf{u}}=\lim _{\varepsilon _k\rightarrow 0}{\textbf{u}}_{\varepsilon _k}\) by establishing two modulated energy inequalities. Moreover, the derivation of the anchoring boundary condition of \({\textbf{u}}\) (see (1.18c) below) uses a blow-up argument, which is inspired by a recent work of Lin–Wang [43]. There the authors have studied isotropic-nematic phase transitions in the static case based on an anisotropic Ericksen’s model.

To state the main result, we assume that

$$\begin{aligned} \begin{aligned} I=\bigcup _{t\in [0,T]} I_t \times \{t\}~ \text { is a smoothly evolving} \\ (d-1)\text {-dimensional submanifold in}~\Omega , \end{aligned} \end{aligned}$$
(1.4)

starting from a \((d-1)\)-dimensional submanifold \(I_0\subset \Omega \). Here a \((d-1)\)-submanifold refers to an embedded closed smooth surface when \(d=3\) and curve when \(d=2\).

Let \(\Omega ^+_t\) be the domain enclosed by \(I_t\), and \(d_I(x,t)\) be the signed-distance from x to \(I_t\) which takes negative values in \(\Omega ^-_t\), and positive values in \(\Omega ^+_t=\Omega \backslash \overline{\Omega ^-_t}\). Equivalently,

$$\begin{aligned} \Omega ^\pm _t:= \{x\in \Omega \mid d_I(x,t)\gtrless 0\}. \end{aligned}$$
(1.5)

For \(\delta >0\), the (open) \(\delta \)-neighborhood of \(I_t\) is denoted by

$$\begin{aligned} B_\delta (I_t):= \{x\in \Omega \mid | d_I(x,t)|<\delta \}. \end{aligned}$$
(1.6)

Let \(\delta _0\in (0,1)\) be a sufficiently small number so that the nearest point projection

$$\begin{aligned} P_{I}(\cdot ,t): B_{4\delta _0}(I_t) \rightarrow I_t \end{aligned}$$

is smooth for any \(t\in [0,T]\), and that the interface (1.4) stays at least \(4\delta _0\) distant away from the physical boundary \(\partial \Omega \). A further description of the geometry can be found in Sect. 2.2 or in [12].

The first step to study the singular limit of (1.2) is to construct a modulated energy which encodes a distance between the energy in (1.1) and an energy corresponding to the moving interface \(I_t\) in (1.4). Following [23, 24, 37], we define an extension of the inward normal vector \({\textbf{n}}(\cdot ,t)\) of \(I_t\) by

$$\begin{aligned} \varvec{\xi }(x,t):=\phi \left( \frac{d_I(x,t)}{\delta _0}\right) \nabla d_I(x,t)\quad \text { for } x\in \Omega , \end{aligned}$$

where \(\phi \in C_c^2( {\mathbb {R}}; [0,1]) \) is an appropriate cut-off function (see (2.11) below for its precise definition). Now we introduce

$$\begin{aligned} E_\varepsilon [{\textbf{u}}_\varepsilon | I](t):=&\int _\Omega \frac{\varepsilon }{2} \mu |{\text {div}}{\textbf{u}}_\varepsilon (\cdot ,t)|^2\, dx\nonumber \\&+\int _\Omega \left( \frac{\varepsilon }{2}\left| \nabla {\textbf{u}}_\varepsilon (\cdot ,t)\right| ^2+\frac{1}{\varepsilon } {F ({\textbf{u}}_\varepsilon (\cdot ,t))}- \varvec{\xi }\cdot \nabla \psi _\varepsilon (\cdot ,t) \right) \, dx, \end{aligned}$$
(1.7)

where \(\psi _\varepsilon \) is defined by

$$\begin{aligned} \psi _\varepsilon (x,t):= \int _0^{| {\textbf{u}}_\varepsilon (x,t)|} g(s)\, ds . \end{aligned}$$
(1.8)

We shall work with a class of potentials \(F({\textbf{u}})\) under standard assumptions (see e.g. [11, 34]). That is,

$$\begin{aligned} F({\textbf{u}}) =f(|{\textbf{u}}|)= g^2(|{\textbf{u}}|)/2, \end{aligned}$$
(1.9)

where f is a double equal-well potential, namely,

$$\begin{aligned}&f\in C^\infty ( {\mathbb {R}}_{\geqslant 0}),\quad f(s)>0\text { for } s\in {\mathbb {R}}_{\geqslant 0}\backslash \{0,1\}, \end{aligned}$$
(1.10a)
$$\begin{aligned}&g\geqslant 0\text { and is {locally} Lipschitz continuous}, ~g(0)=g(1)=0. \end{aligned}$$
(1.10b)

Moreover, the following structural assumptions on f are made:

$$\begin{aligned}&\exists s_0\in (0,1) \text { s.t. } f'(s) >0\text { on } (0,s_0)~\text { and }~ f'(s) <0\text { on } ( s_0,1); \end{aligned}$$
(1.11a)
$$\begin{aligned}&f'(0)= f'(1)=0,\qquad f''(0), f''(1)>0; \end{aligned}$$
(1.11b)
$$\begin{aligned}&\exists c_0\in (0,1) \text { s.t. } 2c_0^2 s^2\leqslant f(s)\leqslant 2c_0^{-2}s^2\text { for any } s\geqslant 100. \end{aligned}$$
(1.11c)

After an appropriate modification for large |s|, the function \(g(s)=|s||s^2-1|\), which corresponds to the Chern–Simons–Higgs potential, satisfies (1.11).

To control the bulk errors, we need another modulated energy:

$$\begin{aligned} B[{\textbf{u}}_\varepsilon | I](t):= \int _\Omega \Big (\sigma \chi -\sigma + 2(\psi _\varepsilon -\sigma )^- \Big )\eta \circ d_I \,\, dx+\int _\Omega \left( \psi _\varepsilon -\sigma \right) ^+|\eta \circ d_I| \, dx. \end{aligned}$$
(1.12)

Here \(\chi (\cdot ,t):={\textbf{1}}_{\Omega _t^+}-{\textbf{1}}_{\Omega _t^-}\), \(h^\pm \) denote the positive/negative parts of a function h respectively, and \(\eta \) is a truncation of the identity function defined by

$$\begin{aligned} \eta (z):=\left\{ \begin{array}{rl} z\qquad \text {when } &{}z \in [-\delta _0, \delta _0],\\ \delta _0\qquad \text {when } &{}z\geqslant \delta _0,\\ -\delta _0\qquad \text {when } &{}z\leqslant -\delta _0. \end{array} \right. \end{aligned}$$
(1.13)

Note that \(( \eta \circ d_I)~\chi \geqslant 0\) in \(\Omega \) due to our convention on the signed-distance function, and thus the two integrands in (1.12) are both non-negative. We refer the readers to the proof of Theorem 4.1 below for more details on the positivity of (1.12).

Now we state the main result of this work:

Theorem 1.1

Let \(d\in \{2,3\}\), and the assumptions (1.10) and (1.11) be in place. Assume that the moving interface I in (1.4) evolves under mean curvature flow, and the initial datum of (1.2) satisfies the following conditions:

$$\begin{aligned}&{{\textbf{u}}_\varepsilon ^{in}\in W^{1,2}_0( \Omega )}, \end{aligned}$$
(1.14a)
$$\begin{aligned}&A_\varepsilon ({\textbf{u}}_\varepsilon ^{in})\leqslant c_1, \end{aligned}$$
(1.14b)
$$\begin{aligned}&E_\varepsilon [{\textbf{u}}_\varepsilon ^{in} | I_0]+B [{\textbf{u}}_\varepsilon ^{in} | I_0]\leqslant c_1\varepsilon , \end{aligned}$$
(1.14c)

where \(c_1>0\) is independent of \(\varepsilon \). Then there exists \(C_1>0\) independent of \(\varepsilon \) such that

$$\begin{aligned}&\sup _{t\in [0,T]}E_\varepsilon [{\textbf{u}}_\varepsilon | I](t)+\sup _{t\in [0,T]}B [{\textbf{u}}_\varepsilon | I](t) \leqslant C_1\varepsilon , \end{aligned}$$
(1.15)
$$\begin{aligned}&\sup _{t\in [0,T]}\int _{\Omega }| \psi _\varepsilon -\sigma {\textbf{1}}_{\Omega _t^+}| \, dx \leqslant C_1\varepsilon ^{1/4}. \end{aligned}$$
(1.16)

Moreover, up to extraction of a subsequence \(\varepsilon _k\downarrow 0\),

$$\begin{aligned} {\textbf{u}}_{\varepsilon _k}\xrightarrow {k\rightarrow \infty } {\textbf{1}}_{\Omega _t^+} {\textbf{u}}~&\text { in }~ C([0,T];L^2_{loc} {(\Omega \backslash I_t)}), \end{aligned}$$
(1.17)

where \({\textbf{u}}\) satisfies the following properties:

$$\begin{aligned}&{\textbf{u}}\in L^\infty (0,T;W^{1,6/5}(\Omega ^+_t;{\mathbb {S}}^2)), ~ \partial _t {\textbf{u}}\in L^2(0,T; L^{6/5}(\Omega ^+_t)), \end{aligned}$$
(1.18a)
$$\begin{aligned}&{\textbf{u}}(x,t)=0 ~\text { for every }~ t\in [0,T]\text { and for a.e. } ~ x\in \Omega _t^-, \end{aligned}$$
(1.18b)
$$\begin{aligned}&({\textbf{u}}\cdot {\textbf{n}})(x,t)=0 \text { for a.e. } t\in [0,T] \text { and for } {\mathcal {H}}^{d-1}\text {-a.e. }x\in I_t. \end{aligned}$$
(1.18c)

Among the conditions in (1.14), the crucial one is (1.14c), which is used to obtain the inequalities in Theorem 3.1 and in Theorem 4.1 below. To construct an initial datum satisfying (1.14), we need the following result.

Proposition 1.1

Let \(I_0\subset \Omega \) be a \((d-1)\)-dimensional submanifold. For any vector field

$$\begin{aligned} {\textbf{u}}^{in}\in W^{1,2}(\Omega ;{\mathbb {S}}^2) ~\text { with } ~ {\textbf{u}}^{in} |_{I_0}\cdot {\textbf{n}}_{I_0}=0~a.e.~\text { on }~I_0, \end{aligned}$$
(1.19)

there exists \({\textbf{u}}_\varepsilon ^{in} \in W^{1,2}_0(\Omega )\cap L^\infty (\Omega )\) such that

$$\begin{aligned} {\left\{ \begin{array}{ll} {\textbf{u}}_\varepsilon ^{in}={\textbf{u}}^{in} &{} \quad \text {in}~ \Omega ^+_0\backslash B_{2\delta _0}(I_0), \\ {\textbf{u}}_\varepsilon ^{in}=0 &{} \quad \text {in } \Omega ^-_0\backslash B_{2\delta _0}(I_0), \end{array}\right. } \end{aligned}$$
(1.20)

and (1.14) holds for a constant \(c_1>0\) which only depends on \(I_0\) and \(\Vert {\textbf{u}}^{in}\Vert _{W^{1,2}(\Omega )}\).

We comment on the conditions in (1.19). When \(d=3\), \(I_0\) is a smooth closed surface in \(\Omega \). Due to topological obstructions, a vector field satisfying (1.19) is usually not smooth. For instance, when \(I_0\) is diffeomorphic to a 2-sphere, due to the hairy ball theorem, \({\textbf{u}}^{in}|_{I_0}\) must have (at least) one pole. One example of such a pole, which is often encountered in the theory of liquid crystal, is given by the hedgehog profile. Locally the tangent vector field near such a pole is \(C^1\)-equivalent to the mapping \({\textbf{h}}(x)=x/|x|: B_1\cap {\mathbb {R}}^2\rightarrow {\mathbb {S}}^1\). Note that \({\textbf{h}}\in W^{\frac{1}{2},2}(B_1\cap {\mathbb {R}}^2)\) but \({\textbf{h}}\notin W^{1,2}(B_1\cap {\mathbb {R}}^2)\). When \(d=2\), there are fewer constraints to arrange a vector field \({\textbf{f}}:I_0\mapsto {\mathbb {S}}^2\subset {\mathbb {R}}^3\) that is orthogonal to the planar curve \(I_0\subset {\mathbb {R}}^2\times \{0\}\). In general, using the extension lemma of Hardt–Lin (cf. [42, Lemma 2.2.10]), any tangent vector field \({\textbf{f}}\in W^{\frac{1}{2}, 2}(I_0;{\mathbb {S}}^2)\) has an extension \({\textbf{u}}^{in}\) satisfying (1.19).

An immediate consequence of Theorem 1.1 is the convergence in (1.3). Indeed, it follows from (1.15) and (2.26b) below that \(\int _{\Omega } \frac{\varepsilon }{2} \mu |{\text {div}}{\textbf{u}}|^2\,dx\xrightarrow {\varepsilon \rightarrow 0}0\), and thus such an energy does not contribute to the surface energy in the limit. However, it forces \({\textbf{u}}\) to satisfy the boundary condition (1.18c). Now applying integration by parts to the last term of (1.7), and then using (1.16) and \(\varvec{\xi }|_{\partial \Omega }=0\), we find

$$\begin{aligned}&\lim _{\varepsilon \rightarrow 0} \int _{\Omega } \left( \frac{\varepsilon }{2} |\nabla {\textbf{u}}_\varepsilon |^2+\frac{1}{\varepsilon }F({\textbf{u}}_\varepsilon ) \right) \, dx\nonumber \\ =&\lim _{\varepsilon \rightarrow 0} -\int _\Omega ({\text {div}}\varvec{\xi }) \psi _\varepsilon \,dx=-\sigma \int _{\Omega _t^+}({\text {div}}\varvec{\xi }) \, dx=\sigma {\mathcal {H}}^{d-1}( I_t). \end{aligned}$$
(1.21)

Note that the last step is due to the Green’s formula.

Under additional assumptions, we can show that the limit \({\textbf{u}}\) in (1.17) solves a geometric evolution equation in the bulk region \(\Omega ^+:=\bigcup _{t\in [0,T]}\Omega ^+_t\times \{t\}\).

Theorem 1.2

Let \(d=2\) and the assumptions of Theorem 1.1 be in place. Assume further that

$$\begin{aligned} \begin{aligned}&f(s)= s^2 \text { for } s\leqslant 1/4; f(s)= (s-1)^2 \text { for } s\geqslant 3/4; \\&f(s)\geqslant 1/{16} \text { for } s\in [1/4, 3/4];\\&\sup _{s\in [1/4, 3/4]} |f'(s)|\leqslant 4. \end{aligned} \end{aligned}$$
(1.22)

Then there exists a sufficiently small \(\mu >0\) (independent of \(\varepsilon \)) such that the vector field \({\textbf{u}}\) in (1.17) satisfies

$$\begin{aligned}&\int _\Omega \partial _t {\textbf{u}}\wedge {\textbf{u}}\cdot \varvec{\Psi }\, dx+\int _\Omega (\nabla {\textbf{u}}\wedge {\textbf{u}})\cdot \nabla \varvec{\Psi }\, dx\nonumber \\&=\mu \int _\Omega ({\text {div}}{\textbf{u}}) \Big (({\text {rot}}\varvec{\Psi })\cdot {\textbf{u}}-({\text {rot}}{\textbf{u}})\cdot \varvec{\Psi }\Big )\, dx \end{aligned}$$
(1.23)

for almost every \(t\in (0,T)\) and for every \( \varvec{\Psi }\in C^1_c(\Omega _t^+;{\mathbb {R}}^3)\).

In the above equation \(\wedge \) is the wedge product in \({\mathbb {R}}^3\) and \({\text {rot}}\) is the curl operator. The equation (1.23) is the weak formulation of an Oseen–Frank flow, written as

$$\begin{aligned} \partial _t {\textbf{u}}=\Delta {\textbf{u}}+\mu ( {{\mathbb {I}}_3}-{\textbf{u}}\otimes {\textbf{u}}) \nabla ({\text {div}}{\textbf{u}})+|\nabla {\textbf{u}}|^2{\textbf{u}},\qquad \text { for } t\in (0,T], x\in \Omega _t^+. \end{aligned}$$
(1.24)

It can be verified that when \({\textbf{u}}\) is sufficiently regular, then (1.23) implies (1.24). It is worth mentioning that equation of the form (1.24) is the \(L^2\)-gradient flow of the variational problem

$$\begin{aligned} \inf \int _{U} \left( \mu |{\text {div}}{\textbf{u}}|^2+ |\nabla {\textbf{u}}|^2\right) \, dx, \end{aligned}$$
(1.25)

where the infimum is taken among mappings \({\textbf{u}}\in W^{1,2}(U;{\mathbb {S}}^2)\) fulfilling certain boundary conditions on \(\partial U\). Note that (1.25) is a special case of the full Oseen–Frank model (cf. [30]).

This work will be organized as follows: In Sect. 2, we shall adapt the modulated energy method of [24] to the vectorial and anisotropic system (1.2), and then derive a differential inequality, i.e. Proposition 2.1. Such an inequality was previously derived in [40] for a matrix-valued equation. When applied to (1.2), it includes a term which does not have an obvious sign due to the additional \({\text {div}}\) term. This problem will be solved in Sect. 3 during the proof of the inequality in Theorem 3.1. This theorem, which leads to the first part of Theorem 1.1, is a major novelty of the present work, and will be employed in Sect. 4 (see Theorem 4.1) to derive the \(L^1\)-estimate of \(\psi _\varepsilon \) in (1.16). Such an estimate will be used in Lemma 4.3 to identify appropriate level sets of \(\psi _\varepsilon \) which converge to \(I_t\) in certain sense. With this key lemma, we derive in Sect. 5 the anchoring boundary condition (1.18c), and thus finish the proof of Theorem 1.1. Section 6 is devoted to the proof of Theorem 1.2. The proof of Proposition 1.1 is quite similar to the construction given in [40]. We present a proof in Appendix A for the convenience of the readers.

2 Preliminaries

2.1 Notation and conventions

We shall adopt the following conventions throughout the paper. Unless specified otherwise, \(C>0\) is a generic constant whose value might change from line to line, and will depend on the geometry of the interface (1.4) but not on \(\varepsilon \) or \(t\in [0,T]\). For two square matrices A and B, their Frobenius inner product is defined by \(A:B:= {\text {tr}}A^{{\textsf{T}}} B\), which induces the norm \(|A|:=\sqrt{{\text {tr}}A^{{\textsf{T}}} A}\). We shall also use the following notation for a vector-valued function \({\textbf{u}}(x,t)=(u_1(x,t),u_2(x,t),u_3(x,t))\) where

$$\begin{aligned} x={\left\{ \begin{array}{ll} (x_1,x_2,x_3) &{}\text { when }d=3,\\ (x_1,x_2,0) &{}\text { when }d=2. \end{array}\right. } \end{aligned}$$
(2.1)
$$\begin{aligned} \begin{aligned} \partial _0&=\partial _t, \quad \partial _i=\partial _{x_i} \quad 1\leqslant i\leqslant 3,\\ \nabla u_1&=(\partial _1 u_1,\partial _2 u_1,\partial _3 u_1),\quad {\text {div}}{\textbf{u}}=\sum _{i=1}^3\partial _i u_i,\\ {\text {rot}}{\textbf{u}}&=(\partial _2 u_3-\partial _3u_2,\partial _3 u_1-\partial _1u_3,\partial _1 u_2-\partial _2u_1). \end{aligned} \end{aligned}$$
(2.2)

To ease computations when \(d=2\), \(\nabla {\textbf{u}}\) will be understood as the matrix \(\begin{pmatrix} \partial _1 u_1&{}\quad \partial _2 u_1 &{}\quad 0 \\ \partial _1 u_2&{}\quad \partial _2 u_2 &{}\quad 0 \\ \partial _1 u_3&{}\quad \partial _2 u_3 &{}\quad 0 \end{pmatrix} \), and

$$\begin{aligned} \begin{aligned}&\text {any planar vector field is understood as a}\\&\text {3D vector field with vanishing 3rd component.} \end{aligned} \end{aligned}$$
(2.3)

In particular, the latter applies to the normal and the mean curvature vector fields (cf. (2.9) and (2.13) respectively below). For a function of \({\textbf{u}}\), like \(F({\textbf{u}})\), its gradient will be denoted by

$$\begin{aligned} D F=(\partial _{u_1} F,\partial _{u_2} F,\partial _{u_3} F). \end{aligned}$$

We end this section by the following assumptions regarding various constants. Theorem 1.1 will be proved for any fixed constant \(\mu >0\), while Theorem 1.2 is valid for a sufficiently small (fixed) \(\mu \). To simplify the presentation, we shall assume without loss of generality that

$$\begin{aligned} \mu \in (0,1)\text { is a fixed constant}. \end{aligned}$$
(2.4)

Finally we can normalize g (cf. (1.9)) to have

$$\begin{aligned} \sigma := \int _0^1 g(s)\, ds=1. \end{aligned}$$
(2.5)

As the \(L^2\)-gradient flow of (1.1), the system (1.2) enjoys the following energy dissipation law:

$$\begin{aligned} A_\varepsilon ({\textbf{u}}_\varepsilon (\cdot ,{\hat{T}}))+ \int _0^{{\hat{T}}} \int _\Omega \varepsilon |\partial _t {\textbf{u}}_\varepsilon |^2 \,d x d t=A_\varepsilon ({\textbf{u}}_\varepsilon ^{in}(\cdot )) \end{aligned}$$
(2.6)

for arbitrarily large time \({\hat{T}}\). Combining this with the theory of gradient flow and the regularity theory for elliptic system (cf. [4, 45]), one can construct a unique solution to system (1.2) that satisfies

$$\begin{aligned} {\textbf{u}}_\varepsilon \in L^2(0,{\hat{T}};W^{2,2}(\Omega )\cap W^{1,2}_0(\Omega ))\text { and }\partial _t {\textbf{u}}_\varepsilon \in L^2(\Omega \times (0,{\hat{T}})). \end{aligned}$$

So for almost every \({\hat{t}}\in (0,{\hat{T}})\), we have

$$\begin{aligned} {\textbf{u}}_\varepsilon (\cdot ,{\hat{t}})\in W^{2,2}(\Omega )\hookrightarrow W^{1,6}(\Omega )\hookrightarrow C^{0,1/2}({\overline{\Omega }}). \end{aligned}$$

Under the assumption (1.11c), the nonlinearity of (1.2a) has a linear growth. So considering the system with initial datum \({\textbf{u}}_\varepsilon (\cdot ,{\hat{t}})\), and using the Hölder estimates for parabolic system (cf. [51]), we deduce that

$$\begin{aligned} {\textbf{u}}_\varepsilon \text { is a classical solution of }(1.2a) \text { in } \Omega \times (0, {\hat{T}}]. \end{aligned}$$
(2.7)

For initial datum undergoing phase transitions near the initial interface \(I_0\), formal asymptotic analysis suggests that \(\nabla {\textbf{u}}_\varepsilon \) will be singular near \(I_t\). However, the global dissipation law (2.6) is not sufficient to yield the (strong) convergence of \({\textbf{u}}_\varepsilon \), not even in the domain away from \(I_t\). Following a recent work of Fisher et al. [24], we shall establish in this section a differential inequality which modulates the concentration and leads to the compactness of solutions in Sobolev spaces.

2.2 The modulated energy

We first set up the geometry of the moving interface I defined in (1.4). Under a local parametrization \(\varvec{\varphi }_t(s):U\subset {\mathbb {R}}^{d-1}\rightarrow I_t\), the mean curvature flow reads

$$\begin{aligned} \partial _t \varvec{\varphi }_t(s) = \kappa {\textbf{n}} \end{aligned}$$
(2.8)

where \( \kappa =\kappa (\varvec{\varphi }_t(s),t)\) is the mean curvature and \({\textbf{n}}={\textbf{n}}(\cdot ,t): I_t\mapsto {\mathbb {S}}^{d-1}\) is the inward normal vector. For any \(t\in [0,T]\) we assume that the nearest-point projection \(P_I(\cdot ,t):B_{4\delta _0}(I_t)\mapsto I_t\) is smooth for some sufficiently small \(\delta _0\in (0,1)\) which only depends on the geometry of I. Analytically we have \(P_I(x,t) =x-\nabla d_I(x,t) d_I(x,t)\). So for each fixed \(t\in [0,T]\), any point \(x\in B_{4\delta _0}(I_t)\) corresponds to a unique pair (rs) with \(r=d_I(x,t)\) and \(s\in U\), and the identity

$$\begin{aligned} d_I\Big (\varvec{\varphi }_t(s)+r{\textbf{n}}(\varvec{\varphi }_t(s),t), t\Big )= r \end{aligned}$$

holds with independent variables (rst). Differentiating this identity with respect to r and t leads to the following identities:

$$\begin{aligned} \begin{aligned} \nabla d_I(x,t)&= {\textbf{n}}(P_I(x,t),t),\\ -\partial _t d_I(x,t)&=\partial _t \varvec{\varphi }_t(s)\cdot {\textbf{n}}(\varvec{\varphi }_t(s),t)=: V(s,t). \end{aligned} \end{aligned}$$
(2.9)

The significance of these equations is that they extend the normal vector and the normal velocity from \(I_t\) to a neighborhood of it. So we shall also use \({\textbf{n}}\) to denote \(\nabla d_I\) when the latter is smooth. We shall extend \({\textbf{n}}\) to the whole computational domain \(\Omega \) by defining

$$\begin{aligned} \varvec{\xi }(x,t):=\phi \left( \frac{d_I(x,t)}{\delta _0}\right) \nabla d_I(x,t) \end{aligned}$$
(2.10)

where \(\phi : {\mathbb {R}}\mapsto {\mathbb {R}}_+\) is an even, smooth function that decreases on [0, 1], and satisfies

$$\begin{aligned} {\left\{ \begin{array}{ll} \phi (z)>0~&{}\text {for}~|z|< 1, \\ \phi (z)=0~&{}\text {for}~|z|\geqslant 1, \\ 1-4 z^2\leqslant \phi (z)\leqslant 1-\frac{1}{2} z^2~&{}\text {for}~|z|\leqslant 1/2. \end{array}\right. } \end{aligned}$$
(2.11)

To fulfill these requirements, we can simply choose

$$\begin{aligned} \phi (z) ={\left\{ \begin{array}{ll} e^{\frac{1}{z^2-1}+1}~&{}\text {for}~|z|< 1,\\ 0~&{}\text {for}~|z|\geqslant 1. \end{array}\right. } \end{aligned}$$
figure a

We proceed with the extension of the mean curvature. Choosing a cut-off function \(\eta _0(x,t)\) such that

$$\begin{aligned} \eta _0(\cdot ,t)\in C_c^\infty (B_{2\delta _0}(I_t);[0,1])~\text { and }\eta _0\equiv 1\text { in }B_{\delta _0}(I_t), \end{aligned}$$
(2.12)

we constantly extend the inward mean curvature vector by defining

$$\begin{aligned} {{\textbf{H}}(x,t):=\kappa \nabla d_I(x,t)} \quad \text {with}\quad \kappa (x,t)=-\Delta d_I (P_I(x,t))\eta _0(x,t). \end{aligned}$$
(2.13)

These combined with (2.10) imply that

$$\begin{aligned}&({\textbf{n}}\cdot \nabla ){\textbf{H}}=0\text { in }B_{\delta _0}(I_t), \end{aligned}$$
(2.14a)
$$\begin{aligned}&(\varvec{\xi }\cdot \nabla ){\textbf{H}}=0 \text { in }\Omega , \end{aligned}$$
(2.14b)
$$\begin{aligned}&\varvec{\xi }=0 ~\text {and}~{\textbf{H}}=0~\text {on}~\partial \Omega . \end{aligned}$$
(2.14c)

Lemma 2.1

There exists a constant \(C>0\) depending only on the geometry of the interface (1.4) such that the following properties hold for every \(t\in [0,T]\):

$$\begin{aligned} |\nabla \cdot \varvec{\xi }+{\textbf{H}}\cdot \varvec{\xi }| \leqslant C&~ | d_I|\quad \text {in}~B_{\delta _0}(I_t), \end{aligned}$$
(2.15a)
$$\begin{aligned} \partial _t d_I +({\textbf{H}}\cdot \nabla ) d_I&=0\quad \text {in}~B_{\delta _0}(I_t), \end{aligned}$$
(2.15b)
$$\begin{aligned} \partial _t \varvec{\xi }+\left( {\textbf{H}}\cdot \nabla \right) \varvec{\xi }+\left( \nabla {\textbf{H}}\right) ^{{\textsf{T}}} \varvec{\xi }&=0\quad \text {in}~B_{\delta _0}(I_t), \end{aligned}$$
(2.15c)

where \(\nabla {\textbf{H}}:=\{\partial _j H_i\}_{1\leqslant i,j\leqslant 3}\) is a matrix with i being the row index.

Proof

By introducing \(\phi _0(\tau ):=\phi (\frac{\tau }{\delta _0})\), we can rewrite (2.10) as \(\varvec{\xi }=\phi _0 \left( d_I\right) \nabla d_I\). Since \(\phi \) is even, we have \(\phi _0'(0)=0\). This combined with Taylor’s expansion in \(d_I\) implies that

$$\begin{aligned} \nabla \cdot \varvec{\xi }&=|\nabla d_I|^2 \phi _0'(d_I)+\phi _0(d_I)\Delta d_I(x,t)\\&=\qquad \quad O(d_I) +\phi _0 (d_I)\Delta d_I(P_I(x,t),t). \end{aligned}$$

This and (2.13) lead to (2.15a). Using (2.9) and (2.13), we can write (2.8) as the transport equation (2.15b), which leads to the following identities in \(B_{\delta _0}(I_t)\):

$$\begin{aligned} \partial _t \nabla d_I+({\textbf{H}}\cdot \nabla ) \nabla d_I +(\nabla {\textbf{H}})^{{\textsf{T}}} \nabla d_I=0,\\ \partial _t \phi _0(d_I )+ ({\textbf{H}}\cdot \nabla ) \phi _0(d_I)=0. \end{aligned}$$

These two equations together imply (2.15c). \(\square \)

It will be convenient to introduce

$$\begin{aligned} \psi _\varepsilon =\textrm{d}^F \circ {\textbf{u}}_\varepsilon \quad \text { where }\textrm{d}^F ({\textbf{v}}):=\int _0^{|{\textbf{v}}|} g(s)\, ds. \end{aligned}$$
(2.16)

It can be verified using (1.10b) that

$$\begin{aligned} \textrm{d}^F ({\textbf{v}}) \in C^1({\mathbb {R}}^3),\quad \text { and }\quad D \textrm{d}^F ({\textbf{v}}) =0~\text { iff }~{\textbf{v}}\in \{0,{\mathbb {S}}^2\}. \end{aligned}$$
(2.17)

By (1.9) we have

$$\begin{aligned} | D \textrm{d}^F ({\textbf{v}})| = \sqrt{2 F ({\textbf{v}})},\qquad \forall {\textbf{v}}\in {\mathbb {R}}^3. \end{aligned}$$
(2.18)

Recalling (2.7), we have

$$\begin{aligned} \partial _i\psi _\varepsilon (x,t)&= \partial _i {\textbf{u}}_\varepsilon (x,t)\cdot D \textrm{d}^F \left( {\textbf{u}}_\varepsilon (x,t)\right) \quad \text { for any }(x,t)\in \Omega \times (0, T], \end{aligned}$$
(2.19a)
$$\begin{aligned} \nabla \psi _\varepsilon (x,t)&= \nabla |{\textbf{u}}_\varepsilon (x,t)| ~g(|{\textbf{u}}_\varepsilon (x,t)|)\qquad \text { if } ~{\textbf{u}}_\varepsilon (x,t)\ne 0. \end{aligned}$$
(2.19b)

Now we define the phase-field analogues of the normal vector and the mean curvature vector respectively by

$$\begin{aligned} {\textbf{n}}_\varepsilon (x,t)&:={\left\{ \begin{array}{ll} \frac{\nabla \psi _\varepsilon }{|\nabla \psi _\varepsilon |}(x,t)&{}\text { if } \nabla \psi _\varepsilon (x,t)\ne 0,\\ 0&{} \text {otherwise}. \end{array}\right. } \end{aligned}$$
(2.20a)
$$\begin{aligned} {\textbf{H}}_\varepsilon (x,t)&:={\left\{ \begin{array}{ll} -\left( \varepsilon \Delta {\textbf{u}}_\varepsilon -\frac{1}{\varepsilon } D F ({\textbf{u}}_\varepsilon ) \right) \cdot \frac{\nabla {\textbf{u}}_\varepsilon }{\left| \nabla {\textbf{u}}_\varepsilon \right| } &{}\text { if } \nabla {\textbf{u}}_\varepsilon \ne 0,\\ 0&{}\text {otherwise}. \end{array}\right. } \end{aligned}$$
(2.20b)

Note that in (2.20b), the inner product is made with the column vectors of \(\nabla {\textbf{u}}_\varepsilon =(\partial _1 {\textbf{u}}_\varepsilon ,\partial _2 {\textbf{u}}_\varepsilon ,\partial _3 {\textbf{u}}_\varepsilon )\). We deduce from (2.20a) that

$$\begin{aligned} \nabla \psi _\varepsilon =|\nabla \psi _\varepsilon | {\textbf{n}}_\varepsilon \quad \text { for any }(x,t). \end{aligned}$$
(2.21)

Define also the orthogonal projection \(\Pi _{{\textbf{u}}_\varepsilon }\) by

$$\begin{aligned} \Pi _{{\textbf{u}}_\varepsilon } \partial _i {\textbf{u}}_\varepsilon := {\left\{ \begin{array}{ll} \left( \partial _i {\textbf{u}}_\varepsilon \cdot \frac{{\textbf{u}}_\varepsilon }{| {\textbf{u}}_\varepsilon | }\right) \frac{{\textbf{u}}_\varepsilon }{| {\textbf{u}}_\varepsilon | }&{}~\text {if}~ {\textbf{u}}_\varepsilon \ne 0,\\ 0,&{}~\text {otherwise}. \end{array}\right. } \end{aligned}$$
(2.22)

Lemma 2.2

The following equations hold:

$$\begin{aligned}&|\nabla \psi _\varepsilon | = |\Pi _{{\textbf{u}}_\varepsilon } \nabla {\textbf{u}}_\varepsilon | | D \textrm{d}^F ({\textbf{u}}_\varepsilon )|\quad \text { for any }(x,t), \end{aligned}$$
(2.23a)
$$\begin{aligned}&\Pi _{{\textbf{u}}_\varepsilon } \nabla {\textbf{u}}_\varepsilon =\frac{|\nabla \psi _\varepsilon |}{| D \textrm{d}^F ({\textbf{u}}_\varepsilon )|^2} D \textrm{d}^F ({\textbf{u}}_\varepsilon )\otimes {\textbf{n}}_\varepsilon \quad \text { on } \{ x\mid |{\textbf{u}}_\varepsilon |\notin \{0,1\}\}. \end{aligned}$$
(2.23b)

Proof

Concerning (2.23a), it suffices to work with the set \(\{ x\mid |{\textbf{u}}_\varepsilon |\notin \{0,1\}\}\) where \(g(|{\textbf{u}}_\varepsilon |)>0\) (cf. (1.10)), for otherwise the equation will follow from (2.17) and (2.19a). On this set we deduce from (2.17) that \( D \textrm{d}^F ({\textbf{u}}_\varepsilon )=\frac{{\textbf{u}}_\varepsilon }{|{\textbf{u}}_\varepsilon |} g(|{\textbf{u}}_\varepsilon |)\ne 0\), and we can rewrite (2.19a) as

$$\begin{aligned} \partial _i \psi _\varepsilon =\partial _i {\textbf{u}}_\varepsilon \cdot \frac{D \textrm{d}^F ({\textbf{u}}_\varepsilon )}{|D \textrm{d}^F ({\textbf{u}}_\varepsilon )|} |D \textrm{d}^F ({\textbf{u}}_\varepsilon )|=\partial _i {\textbf{u}}_\varepsilon \cdot \frac{{\textbf{u}}_\varepsilon }{| {\textbf{u}}_\varepsilon | } |D \textrm{d}^F ({\textbf{u}}_\varepsilon )|. \end{aligned}$$
(2.24)

This combined with (2.22) implies (2.23a).

Now we turn to the proof of (2.23b). On the set \(\{ x\mid |{\textbf{u}}_\varepsilon |\notin \{0,1\}\}\), we have

$$\begin{aligned} \frac{|\nabla \psi _\varepsilon |}{| D \textrm{d}^F ({\textbf{u}}_\varepsilon )|^2} D \textrm{d}^F ({\textbf{u}}_\varepsilon )\otimes {\textbf{n}}_\varepsilon \overset{(2.21)}{=}\frac{ D \textrm{d}^F ({\textbf{u}}_\varepsilon ) }{| D \textrm{d}^F ({\textbf{u}}_\varepsilon )|^2}\otimes \nabla \psi _\varepsilon \overset{(2.19b)}{=} \frac{{\textbf{u}}_\varepsilon }{| {\textbf{u}}_\varepsilon |}\otimes \nabla |{\textbf{u}}_\varepsilon |, \end{aligned}$$
(2.25)

and this implies (2.23b) in view of (2.22). \(\square \)

The following lemma establishes coercivity properties of the modulated energy (1.7).

Lemma 2.3

The following estimates hold for every \(t\in [0,T]\):

$$\begin{aligned} \int _\Omega \left( \frac{\varepsilon }{2} \left| \nabla {\textbf{u}}_\varepsilon \right| ^2+\frac{1}{\varepsilon } F ({\textbf{u}}_\varepsilon )-|\nabla \psi _\varepsilon | \right) \, d x&\leqslant E_\varepsilon [{\textbf{u}}_\varepsilon | I] , \end{aligned}$$
(2.26a)
$$\begin{aligned} \varepsilon \int _\Omega \left( \mu |{\text {div}}{\textbf{u}}_\varepsilon |^2+\left| \nabla {\textbf{u}}_\varepsilon -\Pi _{{\textbf{u}}_\varepsilon }\nabla {\textbf{u}}_\varepsilon \right| ^2 \right) \, d x&\leqslant 2 E_\varepsilon [{\textbf{u}}_\varepsilon | I] , \end{aligned}$$
(2.26b)
$$\begin{aligned} \int _\Omega \left( \sqrt{\varepsilon }\left| \Pi _{{\textbf{u}}_\varepsilon }\nabla {\textbf{u}}_\varepsilon \right| -\frac{1}{\sqrt{\varepsilon }} \left| D \textrm{d}^F ({\textbf{u}}_\varepsilon )\right| \right) ^{2}\, d x&\leqslant 2 E_\varepsilon [ {\textbf{u}}_\varepsilon | I] , \end{aligned}$$
(2.26c)
$$\begin{aligned} \int _\Omega \left( {\frac{\varepsilon }{2}}\left| \nabla {\textbf{u}}_\varepsilon \right| ^{2} +\frac{1}{\varepsilon } F ({\textbf{u}}_\varepsilon )+\left| \nabla \psi _\varepsilon \right| \right) \left( 1-\varvec{\xi }\cdot {\textbf{n}}_\varepsilon \right) \, d x&\leqslant {4 E_\varepsilon [ {\textbf{u}}_\varepsilon | I]}, \end{aligned}$$
(2.26d)
$$\begin{aligned} \int _\Omega \left( \frac{\varepsilon }{2} \left| \nabla {\textbf{u}}_\varepsilon \right| ^{2} +\frac{1}{\varepsilon } F ({\textbf{u}}_\varepsilon ) +|\nabla \psi _\varepsilon |\right) \min \left( d^2_I,1\right) \, d x&\leqslant C E_\varepsilon [ {\textbf{u}}_\varepsilon | I] \end{aligned}$$
(2.26e)

where \(C=C(\delta _0,\phi )\).

Proof

The case when \(\mu = 0\) has been done in [40], and the proof carries over to the present case. First, it follows from (2.22) that

$$\begin{aligned} \left| \nabla {\textbf{u}}_\varepsilon -\Pi _{{\textbf{u}}_\varepsilon }\nabla {\textbf{u}}_\varepsilon \right| ^2+\left| \Pi _{{\textbf{u}}_\varepsilon }\nabla {\textbf{u}}_\varepsilon \right| ^2=\left| \nabla {\textbf{u}}_\varepsilon \right| ^2. \end{aligned}$$
(2.27)

Combining this with (2.21), we can write

$$\begin{aligned}&\frac{\varepsilon }{2} \left| \nabla {\textbf{u}}_\varepsilon \right| ^2+\frac{1}{\varepsilon } F ({\textbf{u}}_\varepsilon )-\varvec{\xi }\cdot \nabla \psi _\varepsilon \nonumber \\ =&\frac{\varepsilon }{2}\left| \nabla {\textbf{u}}_\varepsilon \right| ^2 +\frac{1}{\varepsilon } F ({\textbf{u}}_\varepsilon )-|\nabla \psi _\varepsilon | + |\nabla \psi _\varepsilon | (1-\varvec{\xi }\cdot {\textbf{n}}_\varepsilon )\nonumber \\ =&\frac{\varepsilon }{2} \left| \nabla {\textbf{u}}_\varepsilon -\Pi _{{\textbf{u}}_\varepsilon }\nabla {\textbf{u}}_\varepsilon \right| ^2 + \left( \frac{\varepsilon }{2}\left| \Pi _{{\textbf{u}}_\varepsilon }\nabla {\textbf{u}}_\varepsilon \right| ^2 +\frac{1}{\varepsilon } F ({\textbf{u}}_\varepsilon )-|\nabla \psi _\varepsilon |\right) \nonumber \\&+ |\nabla \psi _\varepsilon | (1-\varvec{\xi }\cdot {\textbf{n}}_\varepsilon ). \end{aligned}$$
(2.28)

By (2.18) and (2.23a), the second term in the last display is non-negative. Since \(|\varvec{\xi }|\leqslant 1\), we also have (2.26a), (2.26b), (2.26c) and

$$\begin{aligned} E_\varepsilon [ {\textbf{u}}_\varepsilon | I]\geqslant \int _\Omega \left( 1-\varvec{\xi }\cdot {\textbf{n}}_\varepsilon \right) \left| \nabla \psi _\varepsilon \right| \,dx. \end{aligned}$$
(2.29)

Combining (2.29) with (2.26a) and the inequality \( 1-\varvec{\xi }\cdot {\textbf{n}}_\varepsilon \leqslant 2\), we obtain (2.26d). Finally, by (2.11) and \(\delta _0\in (0,1)\) we have

$$\begin{aligned} 1-\varvec{\xi }\cdot {\textbf{n}}_\varepsilon \geqslant 1-\phi \left( \frac{d_I}{\delta _0}\right) \geqslant \min \left( \frac{d^2_I}{2\delta _0^2}, 1-\phi (\tfrac{1}{2})\right) \geqslant C \min (d^2_I,1). \end{aligned}$$
(2.30)

This together with (2.26d) implies (2.26e). \(\square \)

The following result was first proved in [24] for the scalar Allen-Cahn equation, and was generalized to the vectorial case in [40].

Proposition 2.1

There exists a generic constant \(C>0\) depending only on the geometry of the interface (1.4) such that

$$\begin{aligned} \frac{d}{d t} E_\varepsilon [ {\textbf{u}}_\varepsilon | I]&+\frac{1}{2\varepsilon }\int _\Omega \left( \varepsilon ^2 \left| \partial _t {\textbf{u}}_\varepsilon \right| ^2-|{\textbf{H}}_\varepsilon |^2\right) \,dx+\frac{1}{2\varepsilon }\int _\Omega \Big | \varepsilon \partial _t {\textbf{u}}_\varepsilon -(\nabla \cdot \varvec{\xi }) D \textrm{d}^F ({\textbf{u}}_\varepsilon ) \Big |^2\,dx\nonumber \\&+\frac{1}{2\varepsilon }\int _\Omega \Big | {\textbf{H}}_\varepsilon -\varepsilon |\nabla {\textbf{u}}_\varepsilon |{\textbf{H}}\Big |^2\,dx \leqslant CE_\varepsilon [ {\textbf{u}}_\varepsilon | I]\qquad \text { for } t\in (0,T]. \end{aligned}$$
(2.31)

We present a proof of (2.31) in Appendix B for the convenience of the readers.

3 Uniform estimates of solutions

Observe that the second term on the left-hand side of (2.31) does not have an obvious sign. However, we have the following theorem.

Theorem 3.1

Under the assumptions of Theorem 1.1, there exists a constant \(C_0>0\), which depends only on the geometry of the interface (1.4) and \(c_1\) (cf. (1.14c)), such that

$$\begin{aligned} \sup _{t\in [0,T]} \frac{1}{\varepsilon }E_\varepsilon [ {\textbf{u}}_\varepsilon | I]+&\int _0^T\int _\Omega \left( \Big |\partial _t {\textbf{u}}_\varepsilon +({\textbf{H}}\cdot \nabla ) {\textbf{u}}_\varepsilon \Big |^2 + \Big |\partial _t {\textbf{u}}_\varepsilon -\Pi _{{\textbf{u}}_\varepsilon }\partial _t {\textbf{u}}_\varepsilon \Big |^2 \right) \, dxdt \leqslant C_0. \end{aligned}$$
(3.1)

It is worth mentioning that \(C_0\) is independent of \(\mu \). The proof of (3.1) relies on the following lemma.

Lemma 3.2

For any function \(\eta _1\) with \(\eta _1(\cdot ,t)\in C_c(B_{4\delta _0}(I_t);{\mathbb {R}}_{\geqslant 0})\), there exists a universal constant \(C>0\) which is independent of t and \(\varepsilon \) such that

$$\begin{aligned} \int _\Omega \eta _1\Big | \nabla {\textbf{u}}_\varepsilon \left( {\mathbb {I}}_3-{\textbf{n}}\otimes {\textbf{n}}\right) \Big |^2\, dx\leqslant C \varepsilon ^{-1} E_\varepsilon [ {\textbf{u}}_\varepsilon | I](t)\qquad \forall t\in [0,T]. \end{aligned}$$
(3.2)

Proof

On the set \(\{ x\mid g(|{\textbf{u}}_\varepsilon |)>0\}=\{ x\mid |{\textbf{u}}_\varepsilon |\notin \{ 0, 1\}\}\) we can use (2.23b) and (2.23a) to estimate

$$\begin{aligned}&\Big | \Pi _{{\textbf{u}}_\varepsilon } \nabla {\textbf{u}}_\varepsilon ( {{\mathbb {I}}_3}- {\textbf{n}}_\varepsilon \otimes \varvec{\xi }) \Big |^2\\ =&\,\left| \frac{|\nabla \psi _\varepsilon |}{| D \textrm{d}^F ({\textbf{u}}_\varepsilon )|^2} D \textrm{d}^F ({\textbf{u}}_\varepsilon )\otimes ({\textbf{n}}_\varepsilon -\varvec{\xi })\right| ^2 \\ \leqslant&\, {| {\textbf{n}}_\varepsilon -\varvec{\xi }|^2}\left| \Pi _{{\textbf{u}}_\varepsilon } \nabla {\textbf{u}}_\varepsilon \right| ^2\\ \leqslant&\, 2(1-\varvec{\xi }\cdot {\textbf{n}}_\varepsilon )\left| \nabla {\textbf{u}}_\varepsilon \right| ^2. \end{aligned}$$

On the set \(\{ x\mid |{\textbf{u}}_\varepsilon |=0\}\) we have \(\Pi _{{\textbf{u}}_\varepsilon } \nabla {\textbf{u}}_\varepsilon =0\) by the second case in (2.22). On the open set \( \{ x\mid |{\textbf{u}}_\varepsilon |>0\}\supset \{ x\mid |{\textbf{u}}_\varepsilon |=1\}\) we can write \(\Pi _{{\textbf{u}}_\varepsilon } \nabla {\textbf{u}}_\varepsilon = \nabla |{\textbf{u}}_\varepsilon | \otimes \frac{{\textbf{u}}_\varepsilon }{|{\textbf{u}}_\varepsilon |}\) by the first case in (2.22). This combined with [18, Theorem 4.4] implies that \(\Pi _{{\textbf{u}}_\varepsilon } \nabla {\textbf{u}}_\varepsilon =0\) for a.e. \(x\in \{ x\mid |{\textbf{u}}_\varepsilon |=1\}\). Altogether we have shown that

$$\begin{aligned} \begin{aligned} \Big | \Pi _{{\textbf{u}}_\varepsilon } \nabla {\textbf{u}}_\varepsilon ( {{\mathbb {I}}_3}- {\textbf{n}}_\varepsilon \otimes \varvec{\xi }) \Big |^2 \leqslant \, 2(1-\varvec{\xi }\cdot {\textbf{n}}_\varepsilon )\left| \nabla {\textbf{u}}_\varepsilon \right| ^2\quad \text { a.e. in }\Omega . \end{aligned} \end{aligned}$$
(3.3)

This together with (2.26d) implies

$$\begin{aligned} \int _\Omega \Big | \Pi _{{\textbf{u}}_\varepsilon } \nabla {\textbf{u}}_\varepsilon ({\mathbb {I}}_3- {\textbf{n}}_\varepsilon \otimes \varvec{\xi }) \Big |^2\, dx \leqslant C \varepsilon ^{-1} E_\varepsilon [ {\textbf{u}}_\varepsilon | I]. \end{aligned}$$
(3.4)

In \(B_{4\delta _0}(I_t)\) where \({\textbf{n}}=\nabla d_I\), we have the decomposition

$$\begin{aligned} {\mathbb {I}}_3-{\textbf{n}}_\varepsilon \otimes {\textbf{n}}={\mathbb {I}}_3- {\textbf{n}}_\varepsilon \otimes \varvec{\xi }+ {\textbf{n}}_\varepsilon \otimes ( \varvec{\xi }-{\textbf{n}}). \end{aligned}$$
(3.5)

Using (2.10) and (2.11), we can estimate the last term by

$$\begin{aligned}&~| \varvec{\xi }-{\textbf{n}}|^2= |{\textbf{n}}_\varepsilon \otimes ( \varvec{\xi }-{\textbf{n}})|^2\nonumber \\&\leqslant 2| \varvec{\xi }-{\textbf{n}}|=2\left( 1-\phi (\tfrac{d_I}{\delta _0})\right) \leqslant C \min \left( d^2_I ,1\right) . \end{aligned}$$
(3.6)

These inequalities and (2.26e) lead to

$$\begin{aligned} \int _\Omega \eta _1\Big | \Pi _{{\textbf{u}}_\varepsilon } \nabla {\textbf{u}}_\varepsilon ({\mathbb {I}}_3- {\textbf{n}}_\varepsilon \otimes {\textbf{n}}) \Big |^2\, dx\leqslant C \varepsilon ^{-1} E_\varepsilon [ {\textbf{u}}_\varepsilon | I]. \end{aligned}$$
(3.7)

Now using (3.6), (2.26d) and (2.26e) we find

$$\begin{aligned} \int _\Omega \eta _1 | \nabla {\textbf{u}}_\varepsilon |^2 \Big ( |{\textbf{n}}_\varepsilon -\varvec{\xi }|^2+|\varvec{\xi }-{\textbf{n}}|^2\Big )\, dx \leqslant C \varepsilon ^{-1} E_\varepsilon [ {\textbf{u}}_\varepsilon | I]. \end{aligned}$$

The above two estimates together with the formula

$$\begin{aligned} ({\mathbb {I}}_3-{\textbf{n}}\otimes {\textbf{n}})-({\mathbb {I}}_3-{\textbf{n}}_\varepsilon \otimes {\textbf{n}})=({\textbf{n}}_\varepsilon -\varvec{\xi })\otimes {\textbf{n}}+(\varvec{\xi }-{\textbf{n}})\otimes {\textbf{n}}\end{aligned}$$

yield (3.2). \(\square \)

To proceed we need an \(L^3\)-estimate of \({\textbf{u}}_\varepsilon \).

Lemma 3.3

Under the assumption (1.14b), there exists a constant \(C=C(c_1)>0\) such that

$$\begin{aligned}&\sup _{t\in [0,T]} A_\varepsilon ({\textbf{u}}_\varepsilon (\cdot ,t)) + \sup _{t\in [0,T]} \Vert \nabla \psi _\varepsilon (\cdot , t)\Vert _{L^1(\Omega ) } \leqslant C, \end{aligned}$$
(3.8a)
$$\begin{aligned}&\sup _{t\in [0,T]} \Vert {\textbf{u}}_\varepsilon (\cdot , t) \Vert _{L^3(\Omega )}\leqslant ~ C. \end{aligned}$$
(3.8b)

Proof

It follows from (2.18), (2.23a) and the Cauchy–Schwarz inequality that

$$\begin{aligned} A_\varepsilon ({\textbf{u}}_\varepsilon )\geqslant \int _\Omega \left( \frac{\varepsilon }{2}|\Pi _{{\textbf{u}}_\varepsilon } \nabla {\textbf{u}}_\varepsilon |^2+\frac{1}{2\varepsilon } |D \textrm{d}^F({\textbf{u}}_\varepsilon )|^2\right) \, dx\geqslant \int _\Omega |\nabla \psi _\varepsilon |\, dx. \end{aligned}$$

This and (2.6) lead to (3.8a). To prove (3.8b), we first note that if \(|{\textbf{u}}_\varepsilon |> 2\), then

$$\begin{aligned} \psi _\varepsilon =\int _0^2g(z) \, dz+\int _2^{|{\textbf{u}}_\varepsilon |}g(z) \, dz\overset{(1.11c)}{\geqslant }c_0 (|{\textbf{u}}_\varepsilon |^2-4). \end{aligned}$$

This combined with Sobolev’s embedding and \(\psi _\varepsilon |_{\partial \Omega }=0\) (cf. (1.2c)) leads to

$$\begin{aligned} \int _\Omega |{\textbf{u}}_\varepsilon |^3\, dx&\leqslant C+\int _{\{x\in \Omega \mid |{\textbf{u}}_\varepsilon |> 2\}} |{\textbf{u}}_\varepsilon |^3\, dx\nonumber \\&\leqslant C\left( 1+ \Vert \psi _\varepsilon \Vert ^{3/2}_{L^{3/2}(\Omega )}\right) \nonumber \\&\leqslant C\left( 1+ \Vert \nabla \psi _\varepsilon \Vert ^{3/2}_{L^1(\Omega )}\right) . \end{aligned}$$

\(\square \)

Proof of Theorem 3.1

We shall only present the proof in 3D because the 2D case is analogous under the conventions made in Sect. 2.1. We shall employ Einstein summation notation by summing over repeated Latin indices.

We first use (2.31) to get

$$\begin{aligned} \frac{2}{\varepsilon }\frac{d}{d t} E_\varepsilon [ {\textbf{u}}_\varepsilon | I]&+\frac{1}{\varepsilon ^2 }\int _\Omega \left[ \Big (\varepsilon ^2 \left| \partial _t {\textbf{u}}_\varepsilon \right| ^2-|{\textbf{H}}_\varepsilon |^2\Big )+\Big | {\textbf{H}}_\varepsilon -\varepsilon |\nabla {\textbf{u}}_\varepsilon |{\textbf{H}}\Big |^2\right] \,dx\nonumber \\&+\frac{1}{\varepsilon ^2 }\int _\Omega \left| \varepsilon \partial _t {\textbf{u}}_\varepsilon - D \textrm{d}^F ({\textbf{u}}_\varepsilon )(\nabla \cdot \varvec{\xi }) \right| ^2\,dx \leqslant \frac{C}{\varepsilon }E_\varepsilon [ {\textbf{u}}_\varepsilon | I]. \end{aligned}$$
(3.9)

Observe that the orthogonal projection (2.22) is parallel to \(D \textrm{d}^F({\textbf{u}}_\varepsilon )\) when it does not vanish. So we can write

$$\begin{aligned}&\left| \varepsilon \partial _t {\textbf{u}}_\varepsilon - D \textrm{d}^F ({\textbf{u}}_\varepsilon ) (\nabla \cdot \varvec{\xi }) \right| ^2\\ =&\left| \varepsilon \partial _t {\textbf{u}}_\varepsilon -\varepsilon \Pi _{{\textbf{u}}_\varepsilon } \partial _t {\textbf{u}}_\varepsilon \right| ^2+\left| \varepsilon \Pi _{{\textbf{u}}_\varepsilon } \partial _t {\textbf{u}}_\varepsilon - D \textrm{d}^F ({\textbf{u}}_\varepsilon ) (\nabla \cdot \varvec{\xi }) \right| ^2. \end{aligned}$$

Substituting this identity into (3.9) we find

$$\begin{aligned} \frac{2}{\varepsilon }\frac{d}{d t} E_\varepsilon [ {\textbf{u}}_\varepsilon | I]&+\frac{1}{\varepsilon ^2 }\int _\Omega \left[ \Big (\varepsilon ^2 \left| \partial _t {\textbf{u}}_\varepsilon \right| ^2-|{\textbf{H}}_\varepsilon |^2\Big )+\Big | {\textbf{H}}_\varepsilon -\varepsilon |\nabla {\textbf{u}}_\varepsilon |{\textbf{H}}\Big |^2\right] \,dx\nonumber \\&+ \int _\Omega \left| \partial _t {\textbf{u}}_\varepsilon -\Pi _{{\textbf{u}}_\varepsilon }\partial _t {\textbf{u}}_\varepsilon \right| ^2\,dx \leqslant \frac{C}{\varepsilon }E_\varepsilon [ {\textbf{u}}_\varepsilon | I]. \end{aligned}$$
(3.10)

To estimate the second term on the left-hand side, we use (1.2a) and (2.20b) to write

$$\begin{aligned} {\textbf{H}}_\varepsilon =- \varepsilon \Big ( \partial _t {\textbf{u}}_\varepsilon -\mu \nabla {\text {div}}{\textbf{u}}_\varepsilon \Big )\cdot \frac{\nabla {\textbf{u}}_\varepsilon }{|\nabla {\textbf{u}}_\varepsilon |}\quad \text { if } \nabla {\textbf{u}}_\varepsilon \ne 0. \end{aligned}$$
(3.11)

Note that the inner product is made with the column vectors of \(\nabla {\textbf{u}}_\varepsilon =(\partial _1 {\textbf{u}}_\varepsilon ,\partial _2 {\textbf{u}}_\varepsilon ,\partial _3 {\textbf{u}}_\varepsilon )\). Using the above formula, we expand the integrands of (3.10) and find

$$\begin{aligned}&~\varepsilon ^2 \left| \partial _t {\textbf{u}}_\varepsilon \right| ^2-|{\textbf{H}}_\varepsilon |^2+ \Big | {\textbf{H}}_\varepsilon -\varepsilon |\nabla {\textbf{u}}_\varepsilon | {\textbf{H}}\Big |^2\\&\quad =~\varepsilon ^2 \left| \partial _t {\textbf{u}}_\varepsilon \right| ^2+\varepsilon ^2 |{\textbf{H}}|^2 |\nabla {\textbf{u}}_\varepsilon |^2+2\varepsilon ^2 \partial _t {\textbf{u}}_\varepsilon \cdot ({\textbf{H}}\cdot \nabla ) {\textbf{u}}_\varepsilon \\&\qquad -2\varepsilon ^2\mu ~ \nabla ({\text {div}}{\textbf{u}}_\varepsilon ) \cdot ({\textbf{H}}\cdot \nabla ) {\textbf{u}}_\varepsilon \\&\quad = ~\varepsilon ^2|\partial _t {\textbf{u}}_\varepsilon +({\textbf{H}}\cdot \nabla ) {\textbf{u}}_\varepsilon |^2+\varepsilon ^2 \left( |{\textbf{H}}|^2 |\nabla {\textbf{u}}_\varepsilon |^2- |({\textbf{H}}\cdot \nabla ) {\textbf{u}}_\varepsilon |^2 \right) \\&\qquad -2\varepsilon ^2\mu ~\nabla ({\text {div}}{\textbf{u}}_\varepsilon )\cdot ({\textbf{H}}\cdot \nabla ) {\textbf{u}}_\varepsilon . \end{aligned}$$

Note that the second term in the last display is non-negative due to Cauchy-Schwarz’s inequality, and this implies that

$$\begin{aligned}&\int _\Omega |\partial _t {\textbf{u}}_\varepsilon +({\textbf{H}}\cdot \nabla ) {\textbf{u}}_\varepsilon |^2 \, dx\\&\quad \leqslant \frac{1}{\varepsilon ^2} \int _\Omega \left[ \Big (\varepsilon ^2 \left| \partial _t {\textbf{u}}_\varepsilon \right| ^2-|{\textbf{H}}_\varepsilon |^2\Big )+ \Big | {\textbf{H}}_\varepsilon -\varepsilon |\nabla {\textbf{u}}_\varepsilon | {\textbf{H}}\Big |^2\right] \, dx\\&\qquad +2\mu \int _\Omega \nabla ({\text {div}}{\textbf{u}}_\varepsilon )\cdot ({\textbf{H}}\cdot \nabla ) {\textbf{u}}_\varepsilon \, dx. \end{aligned}$$

Adding the above inequality to (3.10) leads to

$$\begin{aligned} 2\varepsilon ^{-1} \frac{d}{d t} E_\varepsilon [ {\textbf{u}}_\varepsilon | I]&+\int _\Omega \Big |\partial _t {\textbf{u}}_\varepsilon +({\textbf{H}}\cdot \nabla ) {\textbf{u}}_\varepsilon \Big |^2 \, dx + \int _\Omega \Big |\partial _t {\textbf{u}}_\varepsilon -\Pi _{{\textbf{u}}_\varepsilon }\partial _t {\textbf{u}}_\varepsilon \Big |^2\,dx \nonumber \\&\leqslant C \varepsilon ^{-1} E_\varepsilon [ {\textbf{u}}_\varepsilon | I]+2\mu \int _\Omega \nabla ({\text {div}}{\textbf{u}}_\varepsilon )\cdot ({\textbf{H}}\cdot \nabla ) {\textbf{u}}_\varepsilon \, dx. \end{aligned}$$
(3.12)

To estimate the last term, we write \({\textbf{u}}_\varepsilon =(u^\varepsilon _i)_{1\leqslant i\leqslant 3}\) and \({\textbf{H}}=(H_i)_{1\leqslant i\leqslant 3}\). Using integration by parts and (2.14c), we obtain

$$\begin{aligned} \begin{aligned}&\int _\Omega \nabla ({\text {div}}{\textbf{u}}_\varepsilon )\cdot ({\textbf{H}}\cdot \nabla ) {\textbf{u}}_\varepsilon \, dx \\&\quad = - \int _\Omega ({\text {div}}{\textbf{u}}_\varepsilon ) ({\textbf{H}}\cdot \nabla ) {\text {div}}{\textbf{u}}_\varepsilon \, dx- \int _\Omega ({\text {div}}{\textbf{u}}_\varepsilon )(\partial _j {\textbf{H}}\cdot \nabla ) u^\varepsilon _j\, dx\\&\quad = \frac{1}{2} \int _\Omega ({\text {div}}{\textbf{H}}) ({\text {div}}{\textbf{u}}_\varepsilon )^2\, dx-\int _\Omega ({\text {div}}{\textbf{u}}_\varepsilon ) \partial _k H_j \partial _k u^\varepsilon _j\, dx \\&\qquad -\int _\Omega ({\text {div}}{\textbf{u}}_\varepsilon ) (\partial _j H_k-\partial _k H_j) \partial _k u^\varepsilon _j \, dx. \end{aligned} \end{aligned}$$
(3.13)

In view of (2.26b), the first integral in the last display of (3.13) is bounded by

$$\begin{aligned} \mu ^{-1} \varepsilon ^{-1} \Vert {\text {div}}{\textbf{H}}\Vert _{L^\infty _{t,x}}E_\varepsilon [ {\textbf{u}}_\varepsilon | I]. \end{aligned}$$

The second integral can be estimated by decomposing \(\nabla u^\varepsilon _j\) and by using (2.14a):

$$\begin{aligned}&-\int _\Omega ({\text {div}}{\textbf{u}}_\varepsilon ) \,\nabla H_j\cdot \nabla u^\varepsilon _j\,dx\nonumber \\&\quad = -\int _\Omega ({\text {div}}{\textbf{u}}_\varepsilon ) \,\nabla H_j\cdot \Big (({\mathbb {I}}_3-{\textbf{n}}\otimes {\textbf{n}}) \nabla u^\varepsilon _j\Big )\,dx-\int _\Omega ({\text {div}}{\textbf{u}}_\varepsilon ) \, ({\textbf{n}}\cdot \nabla H_j) \left( {\textbf{n}}\cdot \nabla u^\varepsilon _j\right) \,dx\nonumber \\&\quad \leqslant \int _\Omega |{\text {div}}{\textbf{u}}_\varepsilon |^2\,dx + \int _\Omega |\nabla {\textbf{H}}|^2 \Big |({\mathbb {I}}_3-{\textbf{n}}\otimes {\textbf{n}}) {\nabla {\textbf{u}}_\varepsilon }\Big |^2\,dx\nonumber \\&\qquad +C \int _\Omega |\nabla {\textbf{u}}_\varepsilon |^2 \min \left( d^2_I ,1\right) \,dx. \end{aligned}$$
(3.14)

By (2.13) and (2.12), the second integral in the last display can be estimated using (3.2) with \(\eta _1:=|\nabla {\textbf{H}}|^2\). The other two terms can be controlled by \( {(\mu ^{-1} +1)}C\varepsilon ^{-1} E_\varepsilon [ {\textbf{u}}_\varepsilon | I]\) using (2.26b) and (2.26e) respectively. To summarize we deduce from (3.13) and (3.14) that

$$\begin{aligned} \begin{aligned}&\int _\Omega \nabla ({\text {div}}{\textbf{u}}_\varepsilon )\cdot ({\textbf{H}}\cdot \nabla ) {\textbf{u}}_\varepsilon \, dx \\&\quad \leqslant ~\mu ^{-1} \varepsilon ^{-1} \Vert {\text {div}}{\textbf{H}}\Vert _{L^\infty _{t,x}}E_\varepsilon [ {\textbf{u}}_\varepsilon | I]+ (\mu ^{-1} +1)C\varepsilon ^{-1} E_\varepsilon [ {\textbf{u}}_\varepsilon | I] \\&\qquad -\int _\Omega ({\text {div}}{\textbf{u}}_\varepsilon ) (\partial _j H_k-\partial _k H_j) \partial _k u^\varepsilon _j \, dx. \end{aligned} \end{aligned}$$

Combining this with (3.12), we find

$$\begin{aligned} 2\varepsilon ^{-1} \frac{d}{d t} E_\varepsilon [ {\textbf{u}}_\varepsilon | I]&+\int _\Omega \Big |\partial _t {\textbf{u}}_\varepsilon +({\textbf{H}}\cdot \nabla ) {\textbf{u}}_\varepsilon \Big |^2 \, dx + \int _\Omega \Big |\partial _t {\textbf{u}}_\varepsilon -\Pi _{{\textbf{u}}_\varepsilon }\partial _t {\textbf{u}}_\varepsilon \Big |^2\,dx \nonumber \\&\leqslant C \varepsilon ^{-1} E_\varepsilon [ {\textbf{u}}_\varepsilon | I]-2\mu \int _\Omega ({\text {div}}{\textbf{u}}_\varepsilon ) (\partial _j H_k-\partial _k H_j) \partial _k u^\varepsilon _j\,dx. \end{aligned}$$
(3.15)

Note that due to (2.4) the constant C above can be made independent of \(\mu \). It remains to estimate the last integral in (3.15). By orthogonal decompositionsFootnote 1,

$$\begin{aligned} (\partial _j H_k-\partial _k H_j) \partial _k u^\varepsilon _j=-( {\text {rot}}{\textbf{u}}_\varepsilon ) \cdot ({\text {rot}}{\textbf{H}}). \end{aligned}$$

We also need the following identity which follows by taking the wedge product of (1.2a) with \({\textbf{u}}_\varepsilon \).

$$\begin{aligned} \mu (\nabla {\text {div}}{\textbf{u}}_\varepsilon ) \wedge {\textbf{u}}_\varepsilon = (\partial _t {\textbf{u}}_\varepsilon - \Delta {\textbf{u}}_\varepsilon ) \wedge {\textbf{u}}_\varepsilon . \end{aligned}$$

Using the above two identities, we integrate by parts to obtain

$$\begin{aligned}&-\mu \int _\Omega ({\text {div}}{\textbf{u}}_\varepsilon ) (\partial _j H_k-\partial _k H_j) \partial _k u^\varepsilon _j\,dx\\&\quad = ~\mu \int _\Omega ({\text {div}}{\textbf{u}}_\varepsilon ) ( {\text {rot}}{\textbf{u}}_\varepsilon ) \cdot ({\text {rot}}{\textbf{H}})\,dx\\&\quad = \mu \int _\Omega ({\text {div}}{\textbf{u}}_\varepsilon ) {\textbf{u}}_\varepsilon \cdot ({\text {rot}}{\text {rot}}{\textbf{H}})\,dx-\int _\Omega \mu (\nabla {\text {div}}{\textbf{u}}_\varepsilon ) \wedge {\textbf{u}}_\varepsilon \cdot ({\text {rot}}{\textbf{H}})\,dx\\&\quad = ~\mu \int _\Omega ({\text {div}}{\textbf{u}}_\varepsilon ) {\textbf{u}}_\varepsilon \cdot ({\text {rot}}{\text {rot}}{\textbf{H}})\,dx -\int _\Omega (\partial _t {\textbf{u}}_\varepsilon - \Delta {\textbf{u}}_\varepsilon ) \wedge {\textbf{u}}_\varepsilon \cdot ({\text {rot}}{\textbf{H}})\,dx\\&\quad = ~\mu \int _\Omega ({\text {div}}{\textbf{u}}_\varepsilon ) {\textbf{u}}_\varepsilon \cdot ({\text {rot}}{\text {rot}}{\textbf{H}})\,dx-\int _\Omega \Big (\partial _t {\textbf{u}}_\varepsilon +({\textbf{H}}\cdot \nabla ) {\textbf{u}}_\varepsilon \Big ) \wedge {\textbf{u}}_\varepsilon \cdot ( {\text {rot}}{\textbf{H}})\,dx\\&\qquad +\int _\Omega ({\textbf{H}}\cdot \nabla ) {\textbf{u}}_\varepsilon \wedge {\textbf{u}}_\varepsilon \cdot ( {\text {rot}}{\textbf{H}})\,dx+\int _\Omega \Delta {\textbf{u}}_\varepsilon \wedge {\textbf{u}}_\varepsilon \cdot ({\text {rot}}{\textbf{H}})\,dx. \end{aligned}$$

Inserting this identity into (3.15), and using the Cauchy–Schwarz inequality, (3.8b) and (2.26b), we find

$$\begin{aligned}&2\varepsilon ^{-1} \frac{d}{d t} E_\varepsilon [ {\textbf{u}}_\varepsilon | I] +\frac{1}{2} \int _\Omega \Big |\partial _t {\textbf{u}}_\varepsilon +({\textbf{H}}\cdot \nabla ) {\textbf{u}}_\varepsilon \Big |^2 \, dx + \int _\Omega \Big |\partial _t {\textbf{u}}_\varepsilon -\Pi _{{\textbf{u}}_\varepsilon }\partial _t {\textbf{u}}_\varepsilon \Big |^2\,dx \nonumber \\&\quad \leqslant C\Big (1+ \varepsilon ^{-1} E_\varepsilon [ {\textbf{u}}_\varepsilon | I]\Big )+2\int _\Omega ({\textbf{H}}\cdot \nabla ) {\textbf{u}}_\varepsilon \wedge {\textbf{u}}_\varepsilon \cdot ( {\text {rot}}{\textbf{H}})\,dx+2\int _\Omega \Delta {\textbf{u}}_\varepsilon \wedge {\textbf{u}}_\varepsilon \cdot ({\text {rot}}{\textbf{H}}) \,dx\nonumber \\&\quad = C\Big (1+ \varepsilon ^{-1} E_\varepsilon [ {\textbf{u}}_\varepsilon | I]\Big )+2\int _\Omega H_k \Big (\partial _k {\textbf{u}}_\varepsilon -\Pi _{{\textbf{u}}_\varepsilon }\partial _k {\textbf{u}}_\varepsilon \Big ) \wedge {\textbf{u}}_\varepsilon \cdot ( {\text {rot}}{\textbf{H}})\,dx\nonumber \\&\qquad \qquad -2\int _\Omega \Big (\partial _k {\textbf{u}}_\varepsilon -\Pi _{{\textbf{u}}_\varepsilon } \partial _k {\textbf{u}}_\varepsilon \Big ) \wedge {\textbf{u}}_\varepsilon \cdot \Big (\partial _k {\text {rot}}{\textbf{H}}\Big )\,dx. \end{aligned}$$
(3.16)

Note that in the last step we used integration by parts, the identity

$$\begin{aligned} (\Pi _{{\textbf{u}}_\varepsilon }\partial _k {\textbf{u}}_\varepsilon )\wedge {\textbf{u}}_\varepsilon =0 \end{aligned}$$
(3.17)

which follows from (2.22), and the identities \(( \partial _k {\textbf{u}}_\varepsilon ) \wedge ( \partial _k {\textbf{u}}_\varepsilon )= 0\) for each fixed \(k\in \{1,2,3\}\). Finally, applying the Cauchy–Schwarz inequality and then (2.26b) and (3.8b) in the last two integrals of (3.16), we find

$$\begin{aligned} 2\varepsilon ^{-1}&\frac{d}{d t} E_\varepsilon [ {\textbf{u}}_\varepsilon | I] +\frac{1}{2} \int _\Omega \Big |\partial _t {\textbf{u}}_\varepsilon +({\textbf{H}}\cdot \nabla ) {\textbf{u}}_\varepsilon \Big |^2 \, dx + \int _\Omega \Big |\partial _t {\textbf{u}}_\varepsilon -\Pi _{{\textbf{u}}_\varepsilon }\partial _t {\textbf{u}}_\varepsilon \Big |^2\,dx \nonumber \\&\quad \leqslant C\Big ( 1+ \varepsilon ^{-1} E_\varepsilon [ {\textbf{u}}_\varepsilon | I]\Big ). \end{aligned}$$
(3.18)

This combined with (1.14c) and Grönwall’s inequality leads to (3.1). \(\square \)

Using (2.26e) and (3.1), we readily obtain the following corollary.

Corollary 3.4

Under the assumptions of Theorem 1.1, there exists a constant \(C>0\), which depends only on the geometry of the interface (1.4) and \(c_1\), such that

$$\begin{aligned} \sup _{t\in [0,T]}\int _{\Omega ^\pm _t\backslash B_{\delta }(I_t)}\left( |\nabla {\textbf{u}}_\varepsilon |^2+\frac{1}{\varepsilon ^2} F({\textbf{u}}_\varepsilon )+\frac{1}{\varepsilon }|\nabla \psi _\varepsilon | \right) \, dx&\leqslant C\delta ^{-2} , \end{aligned}$$
(3.19a)
$$\begin{aligned} \int _0^T\int _{\Omega ^\pm _t\backslash B_{\delta }(I_t)} |\partial _t {\textbf{u}}_\varepsilon |^2\, dx dt&\leqslant C\delta ^{-2}, \end{aligned}$$
(3.19b)

hold for each fixed \(\delta \in (0, \delta _0)\).

Indeed, (3.19b) follows from (3.19a) and the inequality

$$\begin{aligned} \int _0^T\int _\Omega \Big |\partial _t {\textbf{u}}_\varepsilon +({\textbf{H}}\cdot \nabla ) {\textbf{u}}_\varepsilon \Big |^2\, dxdt\leqslant C, \end{aligned}$$
(3.20)

which is a consequence of (3.1). Another consequence of (3.1) is the following lemma concerning

$$\begin{aligned} {\widehat{{\textbf{u}}}}_\varepsilon :={\left\{ \begin{array}{ll} \frac{{\textbf{u}}_\varepsilon }{|{\textbf{u}}_\varepsilon |}&{}\text { if } {\textbf{u}}_\varepsilon \ne 0,\\ 0&{} \text {otherwise}. \end{array}\right. } \end{aligned}$$
(3.21)

Lemma 3.5

Under the assumptions of Theorem 1.1, there exists a constant \(C>0\), which depends only on the geometry of the interface (1.4) and \(c_1\), such that

$$\begin{aligned}&\sup _{t\in [0,T]}\int _\Omega |{\textbf{u}}_\varepsilon |^2 \left| \nabla {\widehat{{\textbf{u}}}}_\varepsilon \right| ^2\, dx+ \sup _{t\in [0,T]}\int _\Omega \Big | {\widehat{{\textbf{u}}}}_\varepsilon \cdot \nabla |{\textbf{u}}_\varepsilon | \Big |^2\, dx\leqslant (1+\mu ^{-1})C , \end{aligned}$$
(3.22a)
$$\begin{aligned}&\sup _{t\in [0,T]}\int _\Omega \left( {\widehat{{\textbf{u}}}}_\varepsilon \cdot {\textbf{n}}_\varepsilon \right) ^2 |\nabla \psi _\varepsilon |\, dx\leqslant (1+\mu ^{-1})(1+\sqrt{\mu +1}) C\varepsilon . \end{aligned}$$
(3.22b)

Proof

We first deduce from (3.1) and (2.26b) that

$$\begin{aligned} \sup _{t\in [0,T]} \int _\Omega \Big ( \mu |{\text {div}}{\textbf{u}}_\varepsilon |^2+\left| \nabla {\textbf{u}}_\varepsilon -\Pi _{{\textbf{u}}_\varepsilon }\nabla {\textbf{u}}_\varepsilon \right| ^2 \Big )\, d x\leqslant C. \end{aligned}$$
(3.23)

By (3.21) we have the identity \({\textbf{u}}_\varepsilon =|{\textbf{u}}_\varepsilon |{\widehat{{\textbf{u}}}}_\varepsilon \). Using this and (2.22), we can write

$$\begin{aligned} \nabla {\textbf{u}}_\varepsilon -\Pi _{{\textbf{u}}_\varepsilon }\nabla {\textbf{u}}_\varepsilon =|{\textbf{u}}_\varepsilon | \nabla {\widehat{{\textbf{u}}}}_\varepsilon ~ \text { if }{\textbf{u}}_\varepsilon \ne 0. \end{aligned}$$
(3.24)

Substituting this formula into (3.23), we obtain the estimate of the first integral on the left-hand side of (3.22a). To control the second one, we use the following formula which follows from (2.22):

$$\begin{aligned} {\text {tr}}\nabla {\textbf{u}}_\varepsilon -{\text {tr}}\left( \Pi _{{\textbf{u}}_\varepsilon }\nabla {\textbf{u}}_\varepsilon \right) ={\text {div}}{\textbf{u}}_\varepsilon -{\widehat{{\textbf{u}}}}_\varepsilon \cdot \nabla |{\textbf{u}}_\varepsilon |~ \text { if }{\textbf{u}}_\varepsilon \ne 0. \end{aligned}$$
(3.25)

Note that on the set \(\{ x\mid |{\textbf{u}}_\varepsilon |=0\}\), we have \(\nabla |{\textbf{u}}_\varepsilon |=0\) a.e., and thus the above formula is still valid. This and (3.23) yield the estimate of \({\widehat{{\textbf{u}}}}_\varepsilon \cdot \nabla |{\textbf{u}}_\varepsilon |\) and (3.22a) is proved.

Regarding (3.22b), it suffices to estimate over the set

$$\begin{aligned} \{ x\mid \nabla \psi _\varepsilon \ne 0\}=:U_\varepsilon \end{aligned}$$

because the integral over its complement vanishes. By (2.17) and (2.19a), we have \(U_\varepsilon \subset \{ x\mid |{\textbf{u}}_\varepsilon |\notin \{0,1\}\}\) where \(g(|{\textbf{u}}_\varepsilon |)=|D \textrm{d}^F|({\textbf{u}}_\varepsilon )>0\). This combined with (2.19b) and (2.20a) implies that

$$\begin{aligned} {\textbf{n}}_\varepsilon =\frac{\nabla \psi _\varepsilon }{|\nabla \psi _\varepsilon |}=\frac{\nabla |{\textbf{u}}_\varepsilon |}{|\nabla |{\textbf{u}}_\varepsilon ||}\quad \text { on } ~U_\varepsilon . \end{aligned}$$

On the other hand, by the polar decomposition \({\textbf{u}}_\varepsilon =|{\textbf{u}}_\varepsilon |{\widehat{{\textbf{u}}}}_\varepsilon \) and orthogonality \({\hat{{\textbf{u}}}}_\varepsilon \perp \partial _{x_j} {\hat{{\textbf{u}}}}_\varepsilon \), we have

$$\begin{aligned} |\nabla {\textbf{u}}_\varepsilon |^2=|\nabla |{\textbf{u}}_\varepsilon | |^2+|{\textbf{u}}_\varepsilon |^2|\nabla {\hat{{\textbf{u}}}}_\varepsilon |^2\geqslant |\nabla |{\textbf{u}}_\varepsilon | |^2\quad \text { on } ~U_\varepsilon . \end{aligned}$$
(3.26)

Setting \({\widehat{{\textbf{u}}}}_\varepsilon \cdot {\textbf{n}}_\varepsilon =:\cos \theta _\varepsilon \), we have

$$\begin{aligned} \mu \int _{U_\varepsilon } \cos ^2 \theta _\varepsilon \big |\nabla |{\textbf{u}}_\varepsilon |\big |^2\, dx=\mu \int _{U_\varepsilon } \left| {\widehat{{\textbf{u}}}}_\varepsilon \cdot {\textbf{n}}_\varepsilon \right| ^2 \big |\nabla |{\textbf{u}}_\varepsilon |\big |^2\, dx\overset{(3.22a)}{\leqslant }(1+\mu ) C. \end{aligned}$$
(3.27)

This inequality, (2.26a) and (3.26) together imply that

$$\begin{aligned} {(1+\mu )}C&\geqslant \int _{U_\varepsilon } \frac{\mu }{2} \cos ^2 \theta _\varepsilon \big |\nabla |{\textbf{u}}_\varepsilon |\big |^2\, dx+\int _{U_\varepsilon } \left( \frac{1}{2} \Big |\nabla |{\textbf{u}}_\varepsilon | \Big |^2+\frac{1}{\varepsilon ^2 } F ({\textbf{u}}_\varepsilon )-\frac{1}{\varepsilon }|\nabla \psi _\varepsilon | \right) \, d x \\&\geqslant \frac{1}{\varepsilon } \int _{U_\varepsilon } \left( \sqrt{ \mu \cos ^2 \theta _\varepsilon +1} \,\,\Big |\nabla |{\textbf{u}}_\varepsilon |\Big |\sqrt{2F ({\textbf{u}}_\varepsilon )}- |\nabla \psi _\varepsilon | \right) \, d x \\&= \frac{1}{\varepsilon } \int _{U_\varepsilon } \left( \sqrt{\mu \cos ^2 \theta _\varepsilon +1} -1\right) |\nabla \psi _\varepsilon | \, d x. \end{aligned}$$

Note that in the last step we have used the identity \(\big |\nabla |{\textbf{u}}_\varepsilon |\big |\sqrt{2F ({\textbf{u}}_\varepsilon )}=|\nabla \psi _\varepsilon |\), which holds on \(U_\varepsilon \). So (3.22b) follows from conjugation. \(\square \)

4 Estimates of level sets

Recalling (2.5), the main result of this section is the following \(L^1\)-estimate of \(\psi _\varepsilon \).

Theorem 4.1

Under the assumptions of Theorem 1.1, there exists \(C>0\) independent of \(\varepsilon \) such that

$$\begin{aligned}&\sup _{t\in [0,T]}B [{\textbf{u}}_\varepsilon | I](t) \leqslant C\varepsilon , \end{aligned}$$
(4.1)
$$\begin{aligned}&\sup _{t\in [0,T]}\int _{\Omega }| \psi _\varepsilon -{\textbf{1}}_{\Omega _t^+}| \, dx \leqslant C\varepsilon ^{1/4}. \end{aligned}$$
(4.2)

Proof

We shall denote the positive and negative parts of a function h by \(h^+\) and \(h^-\) respectively. For simplicity we shall suppress \(\, dx\) in a volume integral. By [18, pp. 153], for any \(h\in W^{1,1}(\Omega )\), we have

$$\begin{aligned} \partial _i (h(x))^+= (\partial _i h(x)) {\textbf{1}}_{\{x\mid h(x)>0\}}(x)\quad \text {for } a.e.~~x\in \Omega . \end{aligned}$$
(4.3)

Our goal is to estimate \(2\psi _\varepsilon -1-\chi \) where \(\chi (x,t)=\pm 1\) in \(\Omega _t^\pm \). Using the formula \(h=h^+-h^-\), we can write

$$\begin{aligned} 2\psi _\varepsilon -1=2(\psi _\varepsilon -1)^++ \left( 1 -2(\psi _\varepsilon -1)^-\right) , \end{aligned}$$
(4.4)

and we shall estimate its difference with \(\chi \). This will be done by establishing differential inequalities for the following energies which add up to (1.12):

$$\begin{aligned} g_\varepsilon (t):=&\int _\Omega ( \psi _\varepsilon -1)^+\zeta \circ d_I , \end{aligned}$$
(4.5a)
$$\begin{aligned} h_\varepsilon (t):=&\int _\Omega \Big (\chi -[1- 2(\psi _\varepsilon -1)^-] \Big )\eta \circ d_I , \end{aligned}$$
(4.5b)

where \(\eta (z)\) is defined by (1.13) and \(|\eta |(z)=:\zeta (z)\). It is obvious that the integrand of (4.5a) is non-negative. Since \(\psi _\varepsilon \geqslant 0\), we have \((\psi _\varepsilon -1)^-\in [0,1]\) and thus \([1 -2(\psi _\varepsilon -1)^-]\) ranges in \([-1,1]\). Using the identity \((\eta \circ d_I)~\chi =|\eta \circ d_I|\), we deduce that the integrand of (4.5b) is also non-negative and

$$\begin{aligned} h_\varepsilon (t)=\int _\Omega \Big |1 -2(\psi _\varepsilon -1)^--\chi \Big | ~\zeta \circ d_I. \end{aligned}$$
(4.6)

Finally, we deduce from (1.14c) that

$$\begin{aligned} g_\varepsilon (0)+h_\varepsilon (0)\leqslant c_1\varepsilon . \end{aligned}$$
(4.7)

Step 1: estimates of weighted errors. Using (1.8) and (1.9), we have

$$\begin{aligned} \partial _t \psi _\varepsilon =&\Big (\partial _t {\textbf{u}}_\varepsilon +({\textbf{H}}\cdot \nabla ){\textbf{u}}_\varepsilon \Big )\cdot \frac{{\textbf{u}}_\varepsilon }{|{\textbf{u}}_\varepsilon |} \sqrt{2F({\textbf{u}}_\varepsilon )}-{\textbf{H}}\cdot \nabla \psi _\varepsilon . \end{aligned}$$
(4.8)

Using this and (4.3) we can calculate

$$\begin{aligned} g_\varepsilon '(t)=&\int _{\{ \psi _\varepsilon> 1\}} (\partial _t {\textbf{u}}_\varepsilon +({\textbf{H}}\cdot \nabla ){\textbf{u}}_\varepsilon )\cdot \frac{{\textbf{u}}_\varepsilon }{|{\textbf{u}}_\varepsilon |} \sqrt{2F({\textbf{u}}_\varepsilon )}~\zeta \circ d_I \\&\qquad -\int _{\{ \psi _\varepsilon> 1\}} {\textbf{H}}\cdot \nabla \psi _\varepsilon ~\zeta \circ d_I +\int _\Omega ( \psi _\varepsilon -1)^+ \partial _t(\zeta \circ d_I) \\ =&\int _{\{ \psi _\varepsilon > 1\}} (\partial _t {\textbf{u}}_\varepsilon +({\textbf{H}}\cdot \nabla ){\textbf{u}}_\varepsilon )\cdot \frac{{\textbf{u}}_\varepsilon }{|{\textbf{u}}_\varepsilon |} \sqrt{2F({\textbf{u}}_\varepsilon )}~\zeta \circ d_I \\&\qquad -\int _\Omega {\textbf{H}}\cdot \nabla ( \psi _\varepsilon -1)^+ ~\zeta \circ d_I -\int _\Omega ( \psi _\varepsilon -1)^+ {\textbf{H}}\cdot \nabla (\zeta \circ d_I) \\&\qquad +\int _\Omega \Big (\partial _t (\zeta \circ d_I)+{\textbf{H}}\cdot \nabla (\zeta \circ d_I)\Big ) ( \psi _\varepsilon -1)^+. \end{aligned}$$

By (2.15b), the integrand of the last integral vanishes on \(B_{\delta _0}(I_t)\). Moreover, we can combine the second and the third integrals in the last display using integration by parts. Using also that \(\Vert {\text {div}}{\textbf{H}}\Vert _{L^\infty _{x,t}}\leqslant C\) and (2.26e), we find

$$\begin{aligned} g_\varepsilon '(t) \leqslant&\int _{\{ \psi _\varepsilon > 1\}} (\partial _t {\textbf{u}}_\varepsilon +({\textbf{H}}\cdot \nabla ){\textbf{u}}_\varepsilon )\cdot \frac{{\textbf{u}}_\varepsilon }{|{\textbf{u}}_\varepsilon |} \sqrt{2F({\textbf{u}}_\varepsilon )}~\zeta \circ d_I\nonumber \\&+\int _\Omega ({\text {div}}{\textbf{H}}) ( \psi _\varepsilon -1)^+ ~\zeta \circ d_I + C\int _{\Omega \backslash B_{\delta _0}(I_t)} ( \psi _\varepsilon -1)^+\nonumber \\ \leqslant&\int _\Omega \varepsilon \Big |\partial _t {\textbf{u}}_\varepsilon +({\textbf{H}}\cdot \nabla ){\textbf{u}}_\varepsilon \Big |^2+ \left( \int _\Omega \frac{1}{\varepsilon }{F({\textbf{u}}_\varepsilon )}\zeta ^2\circ d_I \right) +Cg_\varepsilon \nonumber \\ \leqslant&C E_\varepsilon [{\textbf{u}}_\varepsilon |I] +Cg_\varepsilon + \int _\Omega \varepsilon \Big |\partial _t {\textbf{u}}_\varepsilon +({\textbf{H}}\cdot \nabla ){\textbf{u}}_\varepsilon \Big |^2. \end{aligned}$$
(4.9)

Now using (4.7), (3.20) and (3.1), we can apply the Grönwall lemma and obtain \(\sup _{t\in [0,T]}g_\varepsilon (t)\leqslant C \varepsilon \) for some C which is independent of \(\varepsilon \). Concerning \(h_\varepsilon \), for simplicity we introduce \(w_\varepsilon :=\chi -[1- 2(\psi _\varepsilon -1)^-]\). Using the identity \((\partial _i \chi )~\eta \circ d_I \equiv 0\) (in the sense of distribution), we find

$$\begin{aligned} (\partial _i w_\varepsilon ) ~\eta \circ d_I=(2\partial _i \psi _\varepsilon )~ {\textbf{1}}_{\{\psi _\varepsilon < 1\}}~\eta \circ d_I. \end{aligned}$$
(4.10)

So by the same calculation for \(g_\varepsilon \) we obtain

$$\begin{aligned} h_\varepsilon '(t) =&\int _{\{ \psi _\varepsilon < 1\}} 2(\partial _t {\textbf{u}}_\varepsilon +({\textbf{H}}\cdot \nabla ){\textbf{u}}_\varepsilon )\cdot \frac{{\textbf{u}}_\varepsilon }{|{\textbf{u}}_\varepsilon |} \sqrt{2F({\textbf{u}}_\varepsilon )}~\eta \circ d_I \\&+\int _\Omega ({\text {div}}{\textbf{H}})w_\varepsilon ~\eta \circ d_I +\int _\Omega \Big (\partial _t (\eta \circ d_I)+({\textbf{H}}\cdot \nabla ) \eta \circ d_I\Big ) w_\varepsilon \\ \leqslant&C E_\varepsilon [{\textbf{u}}_\varepsilon |I] +Ch_\varepsilon (t)+\int _\Omega \varepsilon \Big |\partial _t {\textbf{u}}_\varepsilon +({\textbf{H}}\cdot \nabla ){\textbf{u}}_\varepsilon \Big |^2. \end{aligned}$$

Using (4.7) and (3.20), we can apply the Grönwall lemma and obtain \(\sup _{t\in [0,T]}h_\varepsilon (t)\leqslant C\varepsilon \). Finally, by (4.4) and (4.6), we find

$$\begin{aligned}&\int _\Omega |2\psi _\varepsilon -1-\chi |\zeta \circ d_I \nonumber \\&\leqslant \int _\Omega 2(\psi _\varepsilon -1)^+\zeta \circ d_I + \int _\Omega \Big |1 -2(\psi _\varepsilon -1)^--\chi \Big | \zeta \circ d_I \nonumber \\&= 2g_\varepsilon (t)+h_\varepsilon (t)\leqslant C\varepsilon \quad \text { for all }t\in [0,T], \end{aligned}$$
(4.11)

and this proves (4.1).

Step 2: remove the weight. First note that (4.11) implies (4.2) with \(\Omega \) replaced by \(\Omega \backslash B_{\delta _0}(I_t)\). So we shall focus on the estimate on \(B_{\delta _0}(I_t)\). We set \(\chi _\varepsilon :=2\psi _\varepsilon -1\) and abbreviate \(\delta _0\) by \(\delta \). For fixed \(t\in [0,T]\) and \(p\in I_t\) with normal vector \({\textbf{n}}={\textbf{n}}(p)\), applying Hölder’s inequality and Lemma 4.2 below with \(f(r,p,t)=\left| \chi \left( p + r {\textbf{n}}, t\right) -\chi _\varepsilon (p+ r {\textbf{n}}, t )\right| \), we find

$$\begin{aligned}&\left( \int _{B_{\delta }(I_t)}\left| \chi (x,t)-\chi _\varepsilon (x, t)\right| \, dx\right) ^{4/3} \\&\quad = \left( \int _{I_t } \int _{-\delta }^{\delta }f(r,p,t) \, {dr} \, d {\mathcal {H}}^{d-1}(p)\right) ^{4/3} \\&\quad \leqslant C \int _{I_t } \left( \int _{-\delta }^{\delta }f(r,p,t) \, {dr} \right) ^{4/3} d {\mathcal {H}}^{d-1}(p) \\&\quad \overset{(4.12)}{\leqslant }C \int _{I_t }\Vert f(\cdot ,p,t)\Vert _{L^{3/2}(-\delta ,\delta )}\left( \int _{-\delta }^{\delta }f(r,p,t) |r| \, dr\right) ^{1/3} \, d{\mathcal {H}}^{d-1}(p) \\&\quad =~ C\Vert f(\cdot ,t)\Vert _{L^{3/2}(B_{\delta }(I_t))}\left( \int _{I_t }\int _{-\delta }^{\delta }f(r,p,t) |r| \, dr \, d{\mathcal {H}}^{d-1}(p))\right) ^{1/3}. \end{aligned}$$

In view of (1.8) and (1.2c), we have \(\psi _\varepsilon =0\) on \(\partial \Omega \). So by Sobolev’s embedding \(W^{1,1}\hookrightarrow L^{3/2}\) we obtain

$$\begin{aligned}&\left( \int _{B_\delta (I_t)}\left| \chi (x,t)-\chi _\varepsilon (x, t)\right| \, dx\right) ^4 \\&\quad \leqslant \,C \left( \Vert \chi \Vert ^3_{L^{3/2}(\Omega )}+\Vert \chi _\varepsilon \Vert ^3_{L^{3/2}(\Omega )}\right) \int _{\Omega }\zeta \circ d_I~ |\chi _\varepsilon -\chi | \, dx \\&\quad \leqslant C (1+ \Vert \nabla \psi _\varepsilon \Vert ^3_{L^1(\Omega )}) \int _{\Omega } \zeta \circ d_I ~|\chi _\varepsilon -\chi | \, dx \leqslant C \varepsilon . \end{aligned}$$

Note that in the last step we employed (3.8a) and (4.11). This gives the desired estimate in \(B_{\delta _0}(I_t)\) and thus the proof of (4.2) is finished. \(\square \)

Lemma 4.2

For any integrable function \(f:[-\delta ,\delta ]\rightarrow {\mathbb {R}}_{\geqslant 0}\), we have

$$\begin{aligned} \left( \int _{-\delta }^\delta f(r)\, dr \right) ^4 \leqslant {6}\Vert f\Vert ^{3}_{L^{3/2}(-\delta ,\delta )} \int _{-\delta }^\delta |r| f(r)\, dr. \end{aligned}$$
(4.12)

Proof

We write \(x=(x_1,x_2,x_3),y=(y_1,y_2,y_3)\) and \(F(x)=f(x_1)f(x_2)f(x_3)\). By symmetry and the Hölder inequality, we find

$$\begin{aligned} \Vert f\Vert ^6_{L^1(0,\delta )}&=\int _{[0,\delta ]^6} F(x)F(y)\, dxdy\\&=2\int _{[0,\delta ]^6\cap \{(x,y)~\mid x_1+x_2+x_3\leqslant y_1+y_2+y_3\}} F(x)F(y)\, dxdy\\&=2 \int _{[0,\delta ]^3} \left( \int _{[0,\delta ]^3\cap \{x~\mid x_1+x_2+x_3\leqslant y_1+y_2+y_3\}} 1\cdot F(x)\, dx\right) F(y)\,dy\\&\leqslant 2 \int _{[0,\delta ]^3} (y_1+y_2+y_3) \left( \int _{[0,\delta ]^3} F^{3/2}(x)\, dx \right) ^{2/3}F(y)\,dy\\&=6 \Vert f\Vert ^3_{L^{3/2}(0,\delta )}\Vert f\Vert ^2_{L^1(0,\delta )} \int _0^\delta r f(r)\, dr. \end{aligned}$$

\(\square \)

Now we turn to the study of the level sets of \(\psi _\varepsilon \). The main tool is the following estimate, which is a consequence of (2.20a), (2.26d) and (3.1).

$$\begin{aligned}&\sup _{t\in [0,T]}\int _U \Big (|\nabla \psi _\varepsilon |- \varvec{\xi }\cdot \nabla \psi _\varepsilon \Big )\, dx \nonumber \\&\quad = \sup _{t\in [0,T]} \int _U \Big (|\nabla \psi _\varepsilon |- \varvec{\xi }\cdot {\textbf{n}}_\varepsilon |\nabla \psi _\varepsilon |\Big )\, dx \leqslant C\varepsilon , \quad \forall U \text { measurable in }\Omega . \end{aligned}$$
(4.13)

Lemma 4.3

For each \(t\in [0,T]\) there exists a null set \({\mathcal {N}}_t^\varepsilon \subset (0,1/8)\) such that the following holds: for every \(\alpha \in (0,1/8)\backslash {\mathcal {N}}_t^\varepsilon \), there exist

$$\begin{aligned} b_{\varepsilon ,\alpha }(t)\in [1/2-\alpha ,1/2+\alpha ]\quad \text { and }\quad q_{\varepsilon ,\alpha }(t)\in [ 2-\alpha , 2+\alpha ] \end{aligned}$$
(4.14)

such that the sets

$$\begin{aligned} \{x\mid \psi _\varepsilon (x,t) >b_{\varepsilon ,\alpha }(t)\}\text { and } \{x\mid \psi _\varepsilon (x,t) <q_{\varepsilon ,\alpha }(t) \} \end{aligned}$$
(4.15)

are of finite perimeter and

$$\begin{aligned} \Big |{\mathcal {H}}^{d-1}(\{x\mid \psi _\varepsilon (x,t) =b_{\varepsilon ,\alpha }(t)\})-{\mathcal {H}}^{d-1} (I_t)\Big |\leqslant C \varepsilon ^{1/4}\alpha ^{-1}, \end{aligned}$$
(4.16a)
$$\begin{aligned} {\mathcal {H}}^{d-1}(\{x\mid \psi _\varepsilon (x,t) =q_{\varepsilon ,\alpha }(t)\}) \leqslant C \varepsilon ^{1/4}\alpha ^{-1}, \end{aligned}$$
(4.16b)

where \(C>0\) is independent of \(t, \varepsilon \) and \(\alpha \).

Proof

To prove (4.16a), we consider the set

$$\begin{aligned} S_t^{\varepsilon ,\alpha } =\{x\in \Omega \mid |2\psi _\varepsilon (x,t)- 1|\leqslant 2\alpha \}, \qquad \forall \alpha \in (0,1/8). \end{aligned}$$
(4.17)

It follows from the co-area formula of BV function [18, section 5.5] that \( S_t^{\varepsilon ,\alpha }\) has finite perimeter for every \(\alpha \in (0,1/8)\backslash \widetilde{{\mathcal {N}}}_t^\varepsilon \) for some null set \(\widetilde{{\mathcal {N}}}_t^\varepsilon \subset (0,1/8)\). Moreover, by (4.13), we have for every \(\alpha \in (0,1/8)\backslash \widetilde{{\mathcal {N}}}_t^\varepsilon \) that

$$\begin{aligned} C\varepsilon&\geqslant \int _{S_t^{\varepsilon ,\alpha }} \Big (|\nabla \psi _\varepsilon |- \varvec{\xi }\cdot \nabla \psi _\varepsilon \Big )\, dx \nonumber \\&=\int _{\frac{1}{2} -\alpha }^{\frac{1}{2} +\alpha } {\mathcal {H}}^{d-1}\left( \{x\mid \psi _\varepsilon =s\}\right) \, ds-\int _{\partial S_t^{\varepsilon ,\alpha }} \varvec{\xi }\cdot \varvec{\nu }\psi _\varepsilon \, d{\mathcal {H}}^{d-1}+\int _{S_t^{\varepsilon ,\alpha }} ({\text {div}}\varvec{\xi }) \psi _\varepsilon \, dx, \end{aligned}$$
(4.18)

where \(\varvec{\nu }\) is the outward normal of the set \( S_t^{\varepsilon ,\alpha }\), defined on its (measure-theoretic) boundary. Since \(|\varvec{\xi }|\leqslant 1\) on \(\Omega \) and \(\psi _\varepsilon \leqslant 1\) on \(S_t^{\varepsilon ,\alpha }\), we have

$$\begin{aligned} \left| \int _{S_t^{\varepsilon ,\alpha }} ({\text {div}}\varvec{\xi }) \psi _\varepsilon \, dx\right| \leqslant C |S_t^{\varepsilon ,\alpha }|, \end{aligned}$$

where \(|A|={\mathcal {L}}^d (A)\) is the d-Lebesgue measure of a set A. Combining this with (4.18), we find

$$\begin{aligned} \left| \int _{\frac{1}{2} -\alpha }^{\frac{1}{2} +\alpha } {\mathcal {H}}^{d-1}\left( \{x\mid \psi _\varepsilon =s\}\right) \, ds- \int _{\partial S_t^{\varepsilon ,\alpha }} \varvec{\xi }\cdot \varvec{\nu }\psi _\varepsilon \, d{\mathcal {H}}^{d-1}\right| \leqslant C( \varepsilon + |S_t^{\varepsilon ,\alpha }|). \end{aligned}$$
(4.19)

By the divergence theorem, we have

$$\begin{aligned} \int _{\partial S_t^{\varepsilon ,\alpha }} \varvec{\xi }\cdot \varvec{\nu }\psi _\varepsilon \, d{\mathcal {H}}^{d-1}&{=} -\left( \tfrac{1}{2}-\alpha \right) \int _{\{x\mid \psi _\varepsilon < \frac{1}{2}-\alpha \}} ({\text {div}}\varvec{\xi }) \, dx- \left( \tfrac{1}{2} +\alpha \right) \int _{\{x\mid \psi _\varepsilon > \frac{1}{2}+\alpha \}} ({\text {div}}\varvec{\xi }) \, dx,\\ -2\alpha {\mathcal {H}}^{d-1} (I_t)&\overset{(2.10)}{=}\quad \left( \tfrac{1}{2}-\alpha \right) \int _{\Omega _t^-}({\text {div}}\varvec{\xi }) \, dx\qquad \quad +\left( \tfrac{1}{2}+\alpha \right) \int _{\Omega _t^+}({\text {div}}\varvec{\xi }) \, dx. \end{aligned}$$

Inserting these two equations into (4.19), we find

$$\begin{aligned}&\left| \int _{\frac{1}{2} -\alpha }^{\frac{1}{2} +\alpha } {\mathcal {H}}^{d-1}\left( \{x\mid \psi _\varepsilon =s\}\right) \, ds-2\alpha {\mathcal {H}}^{d-1} (I_t)\right| \nonumber \\&\quad \leqslant C\left( \varepsilon + |S_t^{\varepsilon ,\alpha }| + \Big | \Omega _t^- \triangle {\{x\mid \psi _\varepsilon <\tfrac{1}{2}-\alpha \}} \Big |+ \Big | \Omega _t^+\triangle {\{x\mid \psi _\varepsilon >\tfrac{1}{2}+\alpha \}}\Big |\right) , \end{aligned}$$
(4.20)

where \(A\triangle B:=(A-B)\cup (B-A)\) is the symmetric difference of two sets A and B. We first estimate \(r_\varepsilon ^+:= ~\Big | \Omega _t^+\triangle {\{x\mid \psi _\varepsilon >\tfrac{1}{2}+\alpha \}}\Big |\).

$$\begin{aligned} r_\varepsilon ^+=&~\Big | \Omega _t^+- {\{x\mid \psi _\varepsilon>\tfrac{1}{2}+\alpha \}}\Big |+\Big | {\{x\mid \psi _\varepsilon>\tfrac{1}{2}+\alpha \}}- \Omega _t^+\Big |\\ =&~\Big | \left( \Omega _t^+- \{x\in \Omega _t^+\mid \psi _\varepsilon>\tfrac{1}{2}+\alpha \} \right) - \{x\in \Omega _t^-\mid \psi _\varepsilon>\tfrac{1}{2}+\alpha \}\Big |\\&+\Big | {\{x\in \Omega _t^-\mid \psi _\varepsilon>\tfrac{1}{2}+\alpha \}}\Big |\\ \leqslant&~\Big | {\{x\in \Omega _t^+\mid \psi _\varepsilon \leqslant \tfrac{1}{2}+\alpha \}}\Big |+ \Big | {\{x\in \Omega _t^-\mid \psi _\varepsilon >\tfrac{1}{2}+\alpha \}}\Big |. \end{aligned}$$

Now using Chebyshev’s inequality and (4.2), we find \(r_\varepsilon ^+\leqslant C \varepsilon ^{1/4} \). Similar estimates apply to \( |S_t^{\varepsilon ,\alpha }|\) and \(r_\varepsilon ^-:= | \Omega _t^- \triangle {\{x\mid \psi _\varepsilon <\tfrac{1}{2}-\alpha \}} |\). Substituting these estimates into (4.20), we find

$$\begin{aligned}&\left| \frac{1}{ 2\alpha } \int _{\frac{1}{2} -\alpha }^{\frac{1}{2} +\alpha } \Big ({\mathcal {H}}^{d-1}\left( \{x\mid \psi _\varepsilon =s\}\right) -{\mathcal {H}}^{d-1} (I_t)\Big )\, ds\right| \leqslant C\varepsilon ^{1/4}\alpha ^{-1}. \end{aligned}$$
(4.21)

So (4.16a) follows from Fubini’s theorem.

To prove (4.16b), we consider the set

$$\begin{aligned} Q_t^{\varepsilon ,\alpha } =\{x\in \Omega \mid |\psi _\varepsilon (x,t)- 2|\leqslant \alpha \}, \qquad \forall \alpha \in (0,1/8), \end{aligned}$$
(4.22)

Using (4.13) and the co-area formula, we have for every \(\alpha \in (0,1/8)\backslash {\mathcal {N}}_t^\varepsilon \) that

$$\begin{aligned} C\varepsilon&\geqslant \int _{Q_t^{\varepsilon ,\alpha }} \left( |\nabla \psi _\varepsilon |- \varvec{\xi }\cdot \nabla \psi _\varepsilon \right) \, dx \\&=\int _{ 2 -\alpha }^{ 2 +\alpha } {\mathcal {H}}^{d-1}\left( \{x\mid \psi _\varepsilon =s\}\right) \, ds-\int _{\partial Q_t^{\varepsilon ,\alpha }} \varvec{\xi }\cdot \varvec{\nu }\psi _\varepsilon \, d{\mathcal {H}}^{d-1}+\int _{Q_t^{\varepsilon ,\alpha }} ({\text {div}}\varvec{\xi }) \psi _\varepsilon \, dx, \end{aligned}$$

where \( {\mathcal {N}}_t^\varepsilon \supset \widetilde{{\mathcal {N}}}_t^\varepsilon \) is a null set in (0, 1/8) and \(\varvec{\nu }\) is the outward normal of \(\partial Q_t^{\varepsilon ,\alpha }\). Since \(\psi _\varepsilon \leqslant 3\) on \(Q_t^{\varepsilon ,\alpha }\), we have \(\left| \int _{Q_t^{\varepsilon ,\alpha }} ({\text {div}}\varvec{\xi }) \psi _\varepsilon \, dx\right| \leqslant C |Q_t^{\varepsilon ,\alpha }|,\) and thus

$$\begin{aligned} \int _{ 2 -\alpha }^{ 2 +\alpha } {\mathcal {H}}^{d-1}\left( \{x\mid \psi _\varepsilon =s\}\right) \, ds\leqslant \left| \int _{\partial Q_t^{\varepsilon ,\alpha }} \varvec{\xi }\cdot \varvec{\nu }\psi _\varepsilon \, d{\mathcal {H}}^{d-1}\right| +C \varepsilon + C |Q_t^{\varepsilon ,\alpha }|. \end{aligned}$$
(4.23)

Using (2.14c), we have \(\int _{\Omega }({\text {div}}\varvec{\xi })\, dx=0\), and thus

$$\begin{aligned}&\int _{\partial Q_t^{\varepsilon ,\alpha }} \varvec{\xi }\cdot \varvec{\nu }\psi _\varepsilon \, d{\mathcal {H}}^{d-1}\\&\quad = \left( 2-\alpha \right) \int _{\{x\mid \psi _\varepsilon \geqslant 2-\alpha \}} ({\text {div}}\varvec{\xi }) \, dx+ \left( 2 +\alpha \right) \int _{\{x\mid \psi _\varepsilon \leqslant 2+\alpha \}} ({\text {div}}\varvec{\xi }) \, dx\\&\quad = \left( 2-\alpha \right) \int _{\{x: \psi _\varepsilon \geqslant 2-\alpha \}} ({\text {div}}\varvec{\xi }) \, dx- \left( 2 +\alpha \right) \int _{\{x\mid \psi _\varepsilon > 2+\alpha \}} ({\text {div}}\varvec{\xi }) \, dx. \end{aligned}$$

This combined with Chebyshev’s inequality and (4.2) implies that

$$\begin{aligned} |Q_t^{\varepsilon ,\alpha }|+\left| \int _{\partial Q_t^{\varepsilon ,\alpha }} \varvec{\xi }\cdot \varvec{\nu }\psi _\varepsilon \, d{\mathcal {H}}^{d-1}\right| \leqslant C \varepsilon ^{1/4}. \end{aligned}$$

Substituting this in (4.23) leads to

$$\begin{aligned} \frac{1}{2\alpha }\int _{ 2 -\alpha }^{ 2 +\alpha } {\mathcal {H}}^{d-1}\left( \{x\mid \psi _\varepsilon =s\}\right) \, ds\leqslant C\varepsilon ^{1/4}\alpha ^{-1}. \end{aligned}$$
(4.24)

So (4.16b) follows from Fubini’s theorem. \(\square \)

We end this section with the following result concerning the convergence of \({\textbf{u}}_{\varepsilon }\).

Proposition 4.1

For every sequence \(\varepsilon _k\downarrow 0\) there exists a subsequence, which we will not relabel, such that \({\textbf{u}}_k:={\textbf{u}}_{\varepsilon _k}\) satisfies

$$\begin{aligned} \partial _t {\textbf{u}}_k\wedge {\textbf{u}}_k \xrightarrow {k\rightarrow \infty }&~\partial _t {\textbf{u}}\wedge {\textbf{u}}~\text {weakly in}~ L^2(0,T;L^{6/5}(\Omega )), \end{aligned}$$
(4.25a)
$$\begin{aligned} \partial _i {\textbf{u}}_k\wedge {\textbf{u}}_k \xrightarrow {k\rightarrow \infty }&~ \partial _i {\textbf{u}}\wedge {\textbf{u}}~\text {weakly-star in}~ L^\infty (0,T;L^{6/5}(\Omega )), ~1\leqslant i\leqslant 3, \end{aligned}$$
(4.25b)

where \({\textbf{u}}={\textbf{u}}(x,t)\) satisfies

$$\begin{aligned} {\textbf{u}}&\in L^\infty (0,T; W^{1,2}_{loc}\cap W^{1,6/5}(\Omega ^+_t;{\mathbb {S}}^2)), \end{aligned}$$
(4.26a)
$$\begin{aligned} \partial _t {\textbf{u}}&\in L^2(0,T; L^2_{loc}\cap L^{6/5}(\Omega ^+_t)), \end{aligned}$$
(4.26b)
$$\begin{aligned} {\textbf{u}}(x,t)&=0 ~ \text { for every }~ t\in [0,T]\text { and for a.e. } x\in \Omega _t^-. \end{aligned}$$
(4.26c)

Furthermore,

$$\begin{aligned} \partial _t {\textbf{u}}_k\xrightarrow { k\rightarrow \infty } \partial _t {\textbf{u}}&~\text {weakly in}~ L^2(0,T;L^2_{loc}(\Omega ^\pm _t)), \end{aligned}$$
(4.27a)
$$\begin{aligned} \nabla {\textbf{u}}_k\xrightarrow {k\rightarrow \infty } \nabla {\textbf{u}}&~\text {weakly-star in}~ L^\infty (0,T;L^2_{loc}(\Omega ^\pm _t)), \end{aligned}$$
(4.27b)
$$\begin{aligned} {\textbf{u}}_k\xrightarrow {k\rightarrow \infty } {\textbf{u}}&~\text {strongly in}~ C([0,T];L^2_{loc}(\Omega ^\pm _t)). \end{aligned}$$
(4.27c)

Before proving this result, we state the Aubin–Lions–Simon lemma. See [41, Theorem 8.62, Exercise 8.63] or [52, Corollary 8] for the proof.

Lemma 4.4

Let \(I \subset {\mathbb {R}}\) be an open bounded interval, let \(\left( Y_0,\Vert \cdot \Vert _{Y_0}\right) ,\left( Y_1,\Vert \cdot \Vert _{Y_1}\right) \), and \(\left( Y_2,\Vert \cdot \Vert _{Y_2}\right) \) be Banach spaces with \(Y_0 \hookrightarrow Y_1 \hookrightarrow Y_2\). Assume that the embedding \(Y_0 \hookrightarrow Y_1\) is compact. Let \({\mathcal {V}}\) be the Banach space of all functions \(u \in L^\infty \left( I; Y_0\right) \) whose distributional derivative \(\partial _t u\) belongs to \(L^2\left( I; Y_2\right) \) endowed with the norm

$$\begin{aligned} \Vert u\Vert _{{\mathcal {V}}}:=\Vert u\Vert _{L^\infty \left( I; Y_0\right) }+ \Vert \partial _t u \Vert _{L^2\left( I; Y_2\right) }. \end{aligned}$$

Then the embedding \({\mathcal {V}} \hookrightarrow C\left( {\bar{I}}; Y_1\right) \) is compact.

Proof of Proposition 4.1

Define \(\Omega ^\pm :=\bigcup _{t\in (0,T]}\Omega _t^\pm \times \{t\}\). We first deduce from (3.1) and (2.26b) that

$$\begin{aligned} \Vert \partial _t {\textbf{u}}_\varepsilon -\Pi _{{\textbf{u}}_\varepsilon }\partial _t {\textbf{u}}_\varepsilon \Vert _{L^2(0,T;L^2(\Omega ))} +\Vert \nabla {\textbf{u}}_\varepsilon -\Pi _{{\textbf{u}}_\varepsilon }\nabla {\textbf{u}}_\varepsilon \Vert _{L^\infty (0,T;L^2(\Omega ))}\leqslant C \end{aligned}$$
(4.28)

for some C independent of \(\varepsilon \). On the other hand, by (2.22) we find

$$\begin{aligned} \Pi _{{\textbf{u}}_\varepsilon }\partial _i {\textbf{u}}_\varepsilon (x,t)\wedge {\textbf{u}}_\varepsilon (x,t)=0\quad \forall (x,t)\in \Omega \times (0,T) \end{aligned}$$
(4.29)

for \(0\leqslant i\leqslant 3\) where \(\partial _0:=\partial _t\). Combining (4.29) and (4.28) with (3.8b), we deduce that

$$\begin{aligned}&\Vert \partial _t {\textbf{u}}_\varepsilon \wedge {\textbf{u}}_\varepsilon \Vert _{L^2(0,T;L^{6/5}(\Omega ))}+\Vert \nabla {\textbf{u}}_\varepsilon \wedge {\textbf{u}}_\varepsilon \Vert _{L^\infty (0,T;L^{6/5}(\Omega ))}\nonumber \\&\quad = \Vert (\partial _t {\textbf{u}}_\varepsilon -\Pi _{{\textbf{u}}_\varepsilon }\partial _t {\textbf{u}}_\varepsilon )\wedge {\textbf{u}}_\varepsilon \Vert _{L^2(0,T;L^{6/5}(\Omega ))}\nonumber \\&\qquad +\Vert (\nabla {\textbf{u}}_\varepsilon -\Pi _{{\textbf{u}}_\varepsilon }\nabla {\textbf{u}}_\varepsilon ) \wedge {\textbf{u}}_\varepsilon \Vert _{L^\infty (0,T;L^{6/5}(\Omega ))}\leqslant C. \end{aligned}$$
(4.30)

So it follows from the Banach–Alaoglu theorem (cf. [41, A.5.]) that

$$\begin{aligned} \begin{aligned} \partial _t {\textbf{u}}_k\wedge {\textbf{u}}_k \xrightarrow {k\rightarrow \infty }&~{\textbf{g}}_0~\text {weakly in}~ L^2(0,T;L^{6/5}(\Omega )),\\ \partial _i {\textbf{u}}_k\wedge {\textbf{u}}_k \xrightarrow {k\rightarrow \infty }&~ {\textbf{g}}_i~\text {weakly-star in}~ L^\infty (0,T;L^{6/5}(\Omega )) \end{aligned} \end{aligned}$$
(4.31)

where

$$\begin{aligned} {\textbf{g}}_0\in L^2(0,T;L^{6/5}(\Omega ))\text { and }\{{\textbf{g}}_i\}_{1\leqslant i\leqslant 3}\subset L^\infty (0,T;L^{6/5}(\Omega )). \end{aligned}$$
(4.32)

It follows from (3.8b), (3.19a) and (3.19b) that, for any fixed \( \delta \in (0,\delta _0)\), up to extraction of subsequences there exists \(\varepsilon _k=\varepsilon _k(\delta )\xrightarrow {k\rightarrow \infty }0\) such that

$$\begin{aligned} {\textbf{u}}_{\varepsilon _k}\xrightarrow {k\rightarrow \infty } {\textbf{u}}&~\text {weakly-star in}~ L^\infty (0,T ;L^3(\Omega )), \end{aligned}$$
(4.33a)
$$\begin{aligned} {\textbf{u}}_{\varepsilon _k}\xrightarrow {k\rightarrow \infty } {\bar{{\textbf{u}}}}_\delta&~\text {weakly-star in}~ L^\infty (0,T ;L^3(\Omega ^\pm _t\backslash B_{\delta }(I_t))), \end{aligned}$$
(4.33b)
$$\begin{aligned} \partial _t {\textbf{u}}_{\varepsilon _k}\xrightarrow { k\rightarrow \infty } \partial _t {\bar{{\textbf{u}}}}_\delta&~\text {weakly in}~ L^2\Big (0,T;L^2(\Omega ^\pm _t\backslash B_{\delta }(I_t))\Big ), \end{aligned}$$
(4.33c)
$$\begin{aligned} \nabla {\textbf{u}}_{\varepsilon _k}\xrightarrow {k\rightarrow \infty } \nabla {\bar{{\textbf{u}}}}_\delta&~\text {weakly-star in}~ L^\infty \Big (0,T;L^2(\Omega ^\pm _t\backslash B_{ \delta }(I_t))\Big ). \end{aligned}$$
(4.33d)

By (4.33a) and (4.33b), we have \({\textbf{u}}={\bar{{\textbf{u}}}}_\delta \) a.e. in \(U^\pm (\delta ):=\cup _{t\in [0,T]} \left( \Omega _t^\pm \backslash B_{\delta }(I_t)\right) \times \{t\}\). This combined with (4.33c) and (4.33d) leads to

$$\begin{aligned} {\textbf{u}}\in L^\infty (0,T;W^{1,2}_{loc}(\Omega ^\pm _t)) ~\text {with}~\partial _t {\textbf{u}}\in L^2(0,T;L^2_{loc}(\Omega ^\pm _t)). \end{aligned}$$
(4.34)

Furthermore, employing (4.33b)–(4.33d) and Lemma 4.4, we obtain

$$\begin{aligned} {\textbf{u}}_{\varepsilon _k}\xrightarrow {k\rightarrow \infty } {\bar{{\textbf{u}}}}_\delta ={\textbf{u}}~\text {strongly in}~ C([0,T];L^2(\Omega ^\pm _t\backslash B_{\delta }(I_t))). \end{aligned}$$
(4.35)

By passing to a sequential limit \(\delta =\delta _\ell \xrightarrow {\ell \rightarrow 0} 0\) and by a diagonal argument we obtain (4.27) up to extraction of subsequences.

Now we turn to the proof of (4.26). Using (3.19a), (4.35) and Fatou’s lemma, we deduce that

$$\begin{aligned} f(|{\textbf{u}}|)=F({\textbf{u}})=F({\bar{{\textbf{u}}}}_\delta )=0\text { a.e. in }U^\pm (\delta ) \end{aligned}$$

for any fixed \(\delta \in (0,\delta _0)\). This together with (1.10) implies that \( {\textbf{u}}\) ranges in \(\{0\}\cup {\mathbb {S}}^2\) a.e. in \(\Omega \times (0,T)\). This combined with (4.2) and (4.34) yields (4.26c) and

$$\begin{aligned} {\textbf{u}}\in L^\infty (0,T;W^{1,2}_{loc}(\Omega ^+_t;{\mathbb {S}}^2)) \text { with }\partial _t {\textbf{u}}\in L^2(0,T;L^2_{loc}(\Omega ^+_t)). \end{aligned}$$
(4.36)

Now we show the integrability of \(\nabla _{x,t} {\textbf{u}}\) up to the boundary. To this aim, we choose a sequence of functions

$$\begin{aligned} \{\eta _k(\cdot ,t)\}_{k\geqslant 1}\subset C_c^\infty (\Omega ^+_t)~\text { with }~\eta _k(\cdot ,t)\xrightarrow {k\rightarrow \infty } {\textbf{1}}_{\Omega ^+_t}\text { in }L^\infty (\Omega ). \end{aligned}$$
(4.37)

By (4.31) and (4.27), we deduce that for \(0\leqslant i\leqslant 3\),

$$\begin{aligned} \eta _k {\textbf{g}}_i= \eta _k \partial _i {\textbf{u}}\wedge {\textbf{u}}~\text { a.e. in } \Omega \times (0,T). \end{aligned}$$
(4.38)

By (4.32) and the dominated convergence theorem, we can take \(k\rightarrow \infty \) and get

$$\begin{aligned} {\textbf{g}}_i=\partial _i {\textbf{u}}\wedge {\textbf{u}}~~ \text { a.e. in }~\Omega \times (0,T),~0\leqslant i\leqslant 3. \end{aligned}$$
(4.39)

This and (4.31) lead to (4.25a) and (4.25b). Since \({\textbf{u}}\) maps \(\Omega ^+\) into \({\mathbb {S}}^2\), we have

$$\begin{aligned} |\partial _i {\textbf{u}}|^2=|\partial _i {\textbf{u}}\wedge {\textbf{u}}|^2=|{\textbf{g}}_i|^2~~a.e.~\text { in }~\Omega ^+,~0\leqslant i\leqslant 3. \end{aligned}$$
(4.40)

This and (4.32) improve (4.36) and yield (4.26a) and (4.26b). \(\square \)

5 Proof of Theorem 1.1: anchoring boundary condition

The inequalities (1.15) and (1.16) have been proved in Theorem 3.1 and in Theorem 4.1. The assertions (1.17), (1.18a) and (1.18b) have been proved in Proposition 4.1 (cf. (4.27c) and (4.26)). It remains to verify (1.18c), and this will be done by applying Lemma 4.3 for every \(t\in [0,T]\) and by choosing an appropriate \(\alpha \) outside the null set \({\mathcal {N}}_t^{\varepsilon _k}\subset (0,1/8)\). For simplicity we shall abbreviate \(\psi _{\varepsilon _k}\) and \({\textbf{u}}_{\varepsilon _k}\) by \(\psi _k\) and \({\textbf{u}}_k\) respectively. For any \(k\geqslant 1\) we can choose \(\beta _k\in [1/2,1]\) such that \(\alpha =\alpha _k:= \beta _k{\varepsilon _k^{1/8}}\notin {\mathcal {N}}_t^{\varepsilon _k}\). Then by Lemma 4.3 there exist

$$\begin{aligned} b_{\varepsilon _k,\alpha _k}(t)=:b_k\in [\tfrac{1}{2}-\alpha _k,\tfrac{1}{2}+\alpha _k],\quad q_{\varepsilon _k,\alpha _k}(t)=:q_k\in [2-\alpha _k,2+\alpha _k] \end{aligned}$$
(5.1)

such that

$$\begin{aligned} (b_k,q_k)\xrightarrow {k\rightarrow \infty }(\tfrac{1}{2},2), \end{aligned}$$
(5.2)

and such that the set

$$\begin{aligned} \Omega _t^k:=\{x\in \Omega \mid b_k<\psi _k(x,t) < q_k\}\text { has finite perimeter}. \end{aligned}$$
(5.3)

Moreover, there exists \(C>0\) which is independent of t and the particular choice of the subsequence \(\varepsilon _k\) such that

$$\begin{aligned}&{\mathcal {H}}^{d-1}(\{x\mid \psi _k(x,t) =q_k\}) \leqslant C {\varepsilon _k^{1/8}}, \end{aligned}$$
(5.4a)
$$\begin{aligned}&\Big |{\mathcal {H}}^{d-1}(\partial \Omega _t^k)-{\mathcal {H}}^{d-1} (I_t)\Big | \leqslant 2 C {\varepsilon _k^{1/8}}. \end{aligned}$$
(5.4b)

Using these level sets, we can prove the following proposition which improves (4.27) to the convergence of \({\textbf{u}}_k\) up to the boundary \(I_t\).

Proposition 5.1

Let \({\textbf{u}}\) be the limit vector field in Proposition 4.1. For a.e. \(t\in [0,T]\), up to extraction of subsequences which we will not relabel, we have

$$\begin{aligned} {\textbf{1}}_{ {\bar{\Omega }}_t^k} {\widehat{{\textbf{u}}}}_k&\xrightarrow {k\rightarrow \infty } {\textbf{1}}_{\Omega _t^+}{\textbf{u}}~\text { weakly-star in } BV(\Omega ), \end{aligned}$$
(5.5a)
$$\begin{aligned} {\textbf{1}}_{ {\bar{\Omega }}_t^k} \nabla {\widehat{{\textbf{u}}}}_k&\xrightarrow {k\rightarrow \infty } {\textbf{1}}_{\Omega _t^+}\nabla {\textbf{u}}~\text { weakly in } L^1(\Omega ), \end{aligned}$$
(5.5b)
$$\begin{aligned} {\textbf{1}}_{ {\bar{\Omega }}_t^k} {\widehat{{\textbf{u}}}}_k&\xrightarrow {k\rightarrow \infty } {\textbf{1}}_{\Omega _t^+}{\textbf{u}}~\text { strongly in } L^p(\Omega ),\text { for any fixed } p\in [1,\infty ), \end{aligned}$$
(5.5c)

where \({\widehat{{\textbf{u}}}}_k={\widehat{{\textbf{u}}}}_{\varepsilon _k}\) is defined in (3.21).

Proof

We first claim that there exists a positive constant \(C_3\) depending only on f (cf. (1.11)) such that the following statement holds for any \(\delta \in (0,1/8)\):

$$\begin{aligned} |{\textbf{u}}_\varepsilon (x,t)|\geqslant C_3\delta \qquad&\qquad \forall x\in \{x:\psi _\varepsilon \geqslant \delta \}. \end{aligned}$$
(5.6)

Indeed, by (1.11a), f (and also g) is increasing on \((0,s_0)\). If \(|{\textbf{u}}_\varepsilon |\geqslant s_0\), we are done. Otherwise,

$$\begin{aligned} \delta \leqslant \psi _\varepsilon =\int ^{|{\textbf{u}}_\varepsilon |}_0g(s) \, ds\leqslant |{\textbf{u}}_\varepsilon | g(s_0), \end{aligned}$$

which implies (5.6). This combined with (3.22a) and (5.3) implies

$$\begin{aligned} \sup _{t\in [0,T]}\int _{\Omega _t^k} \left| \nabla {\widehat{{\textbf{u}}}}_k\right| ^2\, dx\leqslant C \end{aligned}$$
(5.7)

for k sufficiently large. This and (5.3) imply that the distributional derivatives of \( {\textbf{v}}_k(\cdot ,t):= {\textbf{1}}_{ {\bar{\Omega }}_t^k} {\widehat{{\textbf{u}}}}_k(\cdot ,t)\) have no Cantor parts, and the absolute continuous parts \(\{{\textbf{1}}_{ {\bar{\Omega }}_t^k} \nabla {\widehat{{\textbf{u}}}}_k\}_{k\geqslant 1}\) is bounded in \(L^2(\Omega )\). Moreover, their jump parts enjoy the estimate

$$\begin{aligned} \int _{\partial \Omega _t^k} |{\textbf{v}}_k(\cdot ,t)-0|^2\,d{\mathcal {H}}^{d-1}\overset{.(5.4b)}{\leqslant }C, \end{aligned}$$

and \(\{{\textbf{v}}_k(\cdot ,t)\}_{k\geqslant 1}\) is bounded in \(L^\infty (\Omega )\). With these properties, it follows from [2] (or [3, Section 4.1]) that \(\{{\textbf{v}}_k(\cdot ,t)\}_{k\geqslant 1}\) is compact in \(SBV(\Omega )\), the class of special functions of bounded variation on \(\Omega \). More precisely, there exists \({\textbf{v}}(\cdot ,t)\in SBV(\Omega )\) s.t. \({\textbf{v}}_k\rightarrow {\textbf{v}}\) weakly-star in \(BV(\Omega )\) as \(k\rightarrow \infty \), and the absolute continuous part of the gradient \(\nabla ^{a} {\textbf{v}}_k={\textbf{1}}_{ {\bar{\Omega }}_t^k} \nabla {\widehat{{\textbf{u}}}}_k\) converges weakly in \(L^1(\Omega )\) to \(\nabla ^{a} {\textbf{v}}\). To identify \({\textbf{v}}\), we use (4.2) to deduce that \({\textbf{1}}_{{\bar{\Omega }}_t^k}\rightarrow {\textbf{1}}_{\Omega _t^+}\) in \(L^1(\Omega )\) as \(k\rightarrow \infty \). This and (4.27c) yield \({\textbf{v}}(\cdot ,t)={\textbf{1}}_{ \Omega _t^+}~{\textbf{u}}(\cdot ,t)\) a.e. in \(\Omega \), and thus (5.5a) and (5.5b) are proved. Finally by (5.5a), the compact embedding of BV functions and the \(L^\infty \) bound we get (5.5c). \(\square \)

To proceed we define the following measures for Borel sets \(A\subset \Omega \):

$$\begin{aligned} \theta (A)&={\mathcal {H}}^{d-1} (A\cap I_t), \end{aligned}$$
(5.8a)
$$\begin{aligned} \theta _k(A)&=\int _{A\cap \Omega _t^k} |\nabla \psi _k|\, dx. \end{aligned}$$
(5.8b)

Lemma 5.1

For a.e. \(t\in [0,T]\),

$$\begin{aligned} \theta _k \xrightarrow {k\rightarrow \infty } \frac{1}{2} \theta \quad&\text { weakly-star as Radon measures.} \end{aligned}$$
(5.9)

Proof

We define truncation functions

$$\begin{aligned} T_k(s)&=\left\{ \begin{array}{rl} 0\qquad \text {when } &{}s\leqslant b_k,\\ s-b_k \qquad \text {when } &{}b_k\leqslant s\leqslant q_k,\\ q_k-b_k\qquad \text {when } &{}s\geqslant q_k, \end{array} \right. \end{aligned}$$
(5.10)
$$\begin{aligned} T(s)&=\left\{ \begin{array}{rl} 0\qquad \text {when } &{}s\leqslant 1/2,\\ s-1/2 \qquad \text {when } &{} 1/2\leqslant s\leqslant 2,\\ 3/2 \qquad \text {when } &{}s\geqslant 2. \end{array} \right. \end{aligned}$$
(5.11)

By (5.2), we have \(T_k\xrightarrow {k\rightarrow \infty }T\) uniformly on \({\mathbb {R}}\). Moreover,

$$\begin{aligned}&\nabla (T_k\circ \psi _k)=\nabla \psi _k ~{\textbf{1}}_{\Omega _t^k}\qquad \text {a.e. in }\Omega , \end{aligned}$$
(5.12a)
$$\begin{aligned}&T_k\circ \psi _k\xrightarrow {k\rightarrow \infty } \tfrac{1}{2} {\textbf{1}}_{\Omega _t^+}\text { strongly in }L^p(\Omega )\qquad \text {for any fixed}~p\in [1,\infty ). \end{aligned}$$
(5.12b)

Indeed, by (2.7) and (2.17) we know that \(\psi _k(\cdot ,t)\in C^1(\Omega )\). Also by (5.3) we have \(T_k'\circ \psi _k={\textbf{1}}_{\Omega _t^k}\) for a.e. \(x\in \Omega \). Therefore, (5.12a) follows from the chain rule (cf. [26, Proposition 3.24]), while (5.12b) follows from (4.2) and the dominated convergence theorem. By (4.13) we have for any \(g\in C^1_c(\Omega )\) that

$$\begin{aligned} \int _\Omega g\, d\theta _k\overset{(5.8b)}{=}\int _{ \Omega _t^k} g |\nabla \psi _k|\, dx&\overset{(4.13)}{=}O(\varepsilon _k)+ \int _{ \Omega _t^k} g \, \varvec{\xi }\cdot \nabla \psi _k \, dx\\&\overset{(5.12a)}{=}O(\varepsilon _k)+ \int _{\Omega } g \, \varvec{\xi }\cdot \nabla (T_k\circ \psi _k)\, dx \\&~=O(\varepsilon _k)- \int _{\Omega } {\text {div}}(g \varvec{\xi }) ~T_k\circ \psi _k\, dx. \end{aligned}$$

Recalling that \(\varvec{\xi }\) is the inward normal of \(I_t\), we use (5.12b) to pass to the limit in the above equations and obtain

$$\begin{aligned} \lim _{k\rightarrow \infty }\int _\Omega g\, d\theta _k\overset{(5.12b)}{=} -\frac{1}{2} \int _{\Omega _t^+} {\text {div}}(g \varvec{\xi }) \, dx =\frac{1}{2}\int _{I_t} g\, d{\mathcal {H}}^{d-1}\overset{(5.8a)}{=}\frac{1}{2} \int _\Omega g\, d\theta , \end{aligned}$$

for any \(g \in C^1_c(\Omega )\). By approximation, one can pass from \(C^1_c(\Omega )\) to \(C^0_c(\Omega )\), and this proves (5.9). \(\square \)

Now we finish the proof of Theorem 1.1 by verifying (1.18c). The proof here is inspired by the blow-up argument in [43]. See also [25] for the applications of such a method in the study of quasi-convex functionals.

Proof of (1.18c) For any \(x_0 \in I_t\) and any \(R>0\), it follows from (5.5c), (5.12b) and the dominated convergence theorem that

$$\begin{aligned} \lim _{k\rightarrow \infty } \int _{ B_R(x_0) } {\textbf{1}}_{{\hat{\Omega }}_t^k}{\widehat{{\textbf{u}}}}_k \cdot \frac{x-x_0}{|x-x_0|} T_k\circ \psi _k\,dx = \frac{1}{2} \int _{ B_R(x_0) } {\textbf{1}}_{\Omega ^+_t}{\textbf{u}}\cdot \frac{x-x_0}{|x-x_0|} \,dx. \end{aligned}$$

We can use spherical coordinate to rewrite the above two integrals in the form of \(\int _0^R\int _{\partial B_r(x_0)}(\cdot ) \,d{\mathcal {H}}^{d-1}dr\), and then apply Fubini’s theorem. Therefore, there exists \(r_{j} \downarrow 0\) such that for each j we have

$$\begin{aligned} \lim _{k\rightarrow \infty } \int _{\partial B_{r_{j}}(x_0) \cap \Omega _t^k } {\widehat{{\textbf{u}}}}_k \cdot \varvec{\nu }~T_k\circ \psi _k\,d{\mathcal {H}}^{d-1} = \frac{1}{2} \int _{\partial B_{r_{j}}(x_0) \cap \Omega ^+_t} {\textbf{u}}\cdot \varvec{\nu }\, d {\mathcal {H}}^{d-1} \end{aligned}$$
(5.13)

where \(\varvec{\nu }\) is the outward normal of \(\partial B_{r_j}(x_0)\). Moreover, we can arrange \(r_j\) such that \(\theta (\partial B_{r_{j}}(x_0))=0\). This combined with (5.9) implies that

$$\begin{aligned} \lim _{k\rightarrow \infty }\theta _k(B_{r_{j}}(x_0)) = \frac{1}{2} \theta (B_{r_{j}}(x_0)). \end{aligned}$$
(5.14)

To proceed, we use convexity to write, for some \(a_m,c_m\in {\mathbb {R}}\), that

$$\begin{aligned} s^2=\sup _{m\in {\mathbb {N}}^+}\, (a_m s+c_m),\quad \forall s\in {\mathbb {R}}. \end{aligned}$$
(5.15)

(cf. [3, Proposition 2.31]). For \(\theta -a.e. ~x_0\in \textrm{supp}(\theta )=I_t\), we have for each \(m\geqslant 1\) that

$$\begin{aligned} 0&\overset{(3.22b)}{=} \lim _{k\rightarrow \infty } \int _{B_{r_{j}}(x_0)} \left( {\widehat{{\textbf{u}}}}_k\cdot {\textbf{n}}_k \right) ^2\, d\theta _k\nonumber \\&\overset{(5.15)}{\geqslant } \lim _{k\rightarrow \infty } \int _{B_{r_{j}}(x_0)} \left( a_m {\widehat{{\textbf{u}}}}_k\cdot {\textbf{n}}_k +c_m\right) \, d\theta _k\nonumber \\&\overset{(2.20a)}{=} \lim _{k\rightarrow \infty } a_m \int _{B_{r_{j}}(x_0)} {\textbf{1}}_{\Omega _t^k} {\widehat{{\textbf{u}}}}_k\cdot \nabla \psi _k\, dx +c_m \theta _k (B_{r_{j}}(x_0)) \nonumber \\&\overset{(5.12a)}{=} a_m\lim _{k\rightarrow \infty }\int _{B_{r_{j}}(x_0)\cap \Omega _t^k} {\widehat{{\textbf{u}}}}_k\cdot \nabla (T_k\circ \psi _k)\, dx + \theta \left( B_{r_{j}}(x_0)\right) \frac{c_m}{2}. \end{aligned}$$
(5.16)

Note that in the last step we also used (5.14). It remains to compute the integral in the last display of (5.16) under the limit \(k\rightarrow \infty \) for fixed jm. To this aim, we use (5.12a) and integration by parts to find

$$\begin{aligned}&\int _{B_{r_{j}}(x_0)\cap \Omega _t^k} {\widehat{{\textbf{u}}}}_k \cdot \nabla (T_k\circ \psi _k) \, dx\nonumber \\&\quad = \int _{\partial \left( B_{r_{j}}(x_0)\cap \Omega _t^k\right) } ({\widehat{{\textbf{u}}}}_k \cdot \varvec{\nu }) ~ T_k\circ \psi _k \, d{\mathcal {H}}^{d-1}- \int _{B_{r_{j}}(x_0)} {\textbf{1}}_{{\bar{\Omega }}_t^k}({\text {div}}{\widehat{{\textbf{u}}}}_k) ~ T_k\circ \psi _k \, dx\nonumber \\&\quad =: A_k-B_k. \end{aligned}$$
(5.17)

Note that the integrand of \(A_k\) is uniformly bounded in \(L^\infty \). To compute the limit of \(A_k\), we first deduce from (5.10) that \(T_k\circ \psi _k =0\) on the set \(\{x\in \Omega \mid \psi _k(x,t)=b_k\}\) which has finite perimeter (cf. (5.4b)). So we employ (5.3) to find

$$\begin{aligned} A_k=&\int _{\partial B_{r_{j}}(x_0)\cap \Omega _t^k} ({\widehat{{\textbf{u}}}}_k \cdot \varvec{\nu })~ T_k\circ \psi _k \, d{\mathcal {H}}^{d-1}+\int _{ B_{r_{j}}(x_0)\cap \{x|\psi _k=q_k\} } ({\widehat{{\textbf{u}}}}_k \cdot \varvec{\nu }) ~ T_k\circ \psi _k \, d{\mathcal {H}}^{d-1}. \end{aligned}$$
(5.18)

The limit of the first integral is given in (5.13), and that of the second vanishes in the limit \(k\rightarrow \infty \) by (5.4a). So we conclude that

$$\begin{aligned} \lim _{k\rightarrow \infty }A_k= \frac{1}{2} \int _{\partial B_{r_{j}}(x_0) \cap \Omega ^+_t} {\textbf{u}}\cdot \varvec{\nu }\, d {\mathcal {H}}^{d-1}. \end{aligned}$$
(5.19)

Concerning the integral \(B_k\), by (5.5b) the sequence \(\{{\textbf{1}}_{{\bar{\Omega }}_t^k}{\text {div}}{\widehat{{\textbf{u}}}}_k\}_{k\geqslant 1}\) converges weakly in \(L^1(\Omega )\). Moreover, \(\{T_k\circ \psi _k\}_{k\geqslant 1}\) is uniformly bounded in \(L^\infty \), and converges a.e. in \(\Omega \) to \( \tfrac{1}{2} {\textbf{1}}_{\Omega _t^+}\), due to (5.12b). Therefore, applying the Product Limit Theorem (cf. [16] or [49, pp. 169]), we obtain

$$\begin{aligned} \lim _{k\rightarrow \infty }B_k=\frac{1}{2} \int _{ B_{r_{j}}(x_0) \cap \Omega ^{+}_t} ({\text {div}}{\textbf{u}})\, dx. \end{aligned}$$
(5.20)

Using (5.19) and (5.20), we can compute the limit in (5.17) and find

$$\begin{aligned}&\lim _{k\rightarrow \infty }\int _{B_{r_{j}}(x_0)\cap \Omega _t^k} {\widehat{{\textbf{u}}}}_k \cdot \nabla (T_k\circ \psi _k) \, dx \nonumber \\ =&\frac{1}{2} \int _{\partial B_{r_{j}}(x_0) \cap \Omega ^+_t} {\textbf{u}}\cdot \varvec{\nu }\, d {\mathcal {H}}^{d-1} -\frac{1}{2} \int _{\partial \left( B_{r_{j}}(x_0) \cap \Omega ^{+}_t\right) } {\textbf{u}}\cdot \varvec{\nu }\, d {\mathcal {H}}^{d-1}\nonumber \\ =&\frac{1}{2} \int _{ B_{r_{j}}(x_0) \cap \partial \Omega ^+_t} {\textbf{u}}\cdot \varvec{\xi }\, d {\mathcal {H}}^{d-1} \end{aligned}$$
(5.21)

where in the last step we used \(\varvec{\xi }=-\varvec{\nu }\) on \(\partial \Omega ^+_t\). Note that \(\varvec{\xi }\) is the inward normal of \(I_t\) according to (2.10), and \(\Omega _t^+\) is the region enclosed by \(I_t\) with outward normal \(\varvec{\nu }\). Substituting (5.21) into (5.16) and then dividing the resulting inequality by \(\theta \left( B_{r_{j}}(x_0)\right) \) and taking \(j\rightarrow \infty \), we find

$$\begin{aligned} 0&\geqslant \lim _{j \rightarrow \infty } \frac{a_m}{\theta \left( B_{r_{j}}(x_0)\right) }\frac{1}{2} \int _{ B_{r_{j}}(x_0) \cap I_t} {\textbf{u}}\cdot \varvec{\xi }\, d {\mathcal {H}}^{d-1} +\frac{c_m}{2}\nonumber \\&\overset{(5.8a)}{=} \frac{a_m}{2} ({\textbf{u}}\cdot \varvec{\xi })(x_0)+\frac{c_m}{2},\qquad \forall m\in {\mathbb {N}}^+. \end{aligned}$$
(5.22)

This together with (5.15) implies that \(({\textbf{u}}\cdot \varvec{\xi })^2(x_0)=0\) for \({\mathcal {H}}^{d-1}\)-a.e. \(x_0\in I_t\). \(\square \)

6 Proof of Theorem 1.2: Oseen–Frank limit in the bulk

The method here is inspired by [17, 38], which has a 2D nature. We set \( \varvec{\tau }_\varepsilon :=\partial _t {\textbf{u}}_\varepsilon \) and write (1.2a) as

$$\begin{aligned} \varvec{\tau }_\varepsilon&=\mu \nabla ({\text {div}}{\textbf{u}}_\varepsilon ) +\Delta {\textbf{u}}_\varepsilon - \varepsilon ^{-2} D F ({\textbf{u}}_\varepsilon )\,\,\,\text {in}~ \Omega \times (0,T). \end{aligned}$$
(6.1)

By Corollary 3.4 and Proposition 4.1 (cf. (4.27c)), for a.e. \(t_0\in (0,T)\) and for any compact set \(K\subset \subset \Omega _{t_0}^+\), we have

$$\begin{aligned} \int _K |\varvec{\tau }_\varepsilon |^2\, dx+&\int _{K}\left( \frac{1}{2} |\nabla {\textbf{u}}_\varepsilon |^2+\frac{1}{\varepsilon ^2} F( {\textbf{u}}_\varepsilon ) \right) \, dx \leqslant {\hat{c}}^2\quad \text { at }~t=t_0, \end{aligned}$$
(6.2a)
$$\begin{aligned}&{\textbf{u}}_{\varepsilon _k}(\cdot ,t_0)\xrightarrow {k\rightarrow \infty } {\textbf{u}}(\cdot ,t_0)~\text { strongly in } L^2(K), \end{aligned}$$
(6.2b)

where \({\hat{c}}={\hat{c}}(K,t_0)>1\) is independent of \(\mu \) and \(\varepsilon \).

Proposition 6.1

Let K be a compact set of \( \Omega _{t_0}^+\) and assume that (6.2a) and (6.2b) hold. There exists an absolute constant \(\Lambda \in (0,1)\) with the following property: under the assumptions

$$\begin{aligned} {\hat{\epsilon }}< \Lambda / {\hat{c}}^2 ~\text { and }~\mu <\Lambda , \end{aligned}$$
(6.3a)
$$\begin{aligned} B_{2r}(x_0)\subset K~ \text { with }~ r<1~ \text { and }~ \end{aligned}$$
(6.3b)
$$\begin{aligned} \int _{B_{2r}(x_0)}\left( \frac{1}{2} |\nabla {\textbf{u}}_\varepsilon |^2+\frac{1}{\varepsilon ^2} F( {\textbf{u}}_\varepsilon ) \right) \, dx&\leqslant {\hat{\epsilon }}^2~\text { at }~t=t_0, \end{aligned}$$
(6.3c)

there exists a subsequence \(\varepsilon _k\downarrow 0\), which we will not relabel, such that

$$\begin{aligned} \nabla {\textbf{u}}_{\varepsilon _k}(\cdot ,t_0)\xrightarrow {k\rightarrow \infty } \nabla {\textbf{u}}(\cdot ,t_0)\text { strongly in }L^2(B_{r/2}(x_0)). \end{aligned}$$
(6.4)

We shall need the following inequality due to the special choice of f in (1.22):

$$\begin{aligned} |f'(s)|^2\leqslant C_4 f(s),\qquad \forall s\geqslant 0, \end{aligned}$$
(6.5)

for some \(C_4>1\). In the sequel \(C_4>1\) will also be used as a generic constant that might change from line to line due to the use of the Sobolev embeddings and elliptic estimates. Note that \(C_4\) is independent of \(\mu \) and r.

Lemma 6.1

Under the assumptions (6.2a) and (6.3b) with a sufficiently small \({\hat{\epsilon }}\) (defined in (6.11) below), we have

$$\begin{aligned} 3/4\leqslant |{\textbf{u}}_\varepsilon (\cdot ,t_0)| \leqslant 5/4 \text { on } B_r(x_0)\quad \text { for } \varepsilon \leqslant r/4. \end{aligned}$$
(6.6)

Proof

Without loss of generality, we assume \(x_0=0\). For brevity we write \(B_r(0)\) as \(B_r\). Since all arguments are made at \(t=t_0\), we shall suppress the time dependence of \({\textbf{u}}_\varepsilon \).

Step 1. There exists \({\hat{C}}>1\) depending on \({\hat{c}}\) such that for any \(x_1 \in B_r\) we have

$$\begin{aligned} |{\textbf{u}}_\varepsilon (x)-{\textbf{u}}_\varepsilon (y)|\leqslant {\hat{C}} \sqrt{\frac{|x-y|}{\varepsilon }},\quad \forall x,y\in B_{\varepsilon }(x_1). \end{aligned}$$
(6.7)

To prove (6.7), let \({\hat{{\textbf{u}}}}_\varepsilon (z)={\textbf{u}}_\varepsilon (x_1+\varepsilon z): B_2\rightarrow {\mathbb {R}}^3\). Then we can write (6.1) as

$$\begin{aligned} \mu \nabla {\text {div}}{\hat{{\textbf{u}}}}_\varepsilon (z) +\Delta {\hat{{\textbf{u}}}}_\varepsilon (z)=\varepsilon ^2\varvec{\tau }_\varepsilon (x_1+\varepsilon z ) + D F ({\hat{{\textbf{u}}}}_\varepsilon (z) ), \quad z\in B_2. \end{aligned}$$
(6.8)

It follows from (6.2a) and a change of variable that \(\{\varepsilon ^2\varvec{\tau }_\varepsilon (x_1+\varepsilon \cdot )\}_{\varepsilon >0}\) is uniformly bounded in \(L^2(B_2)\). Using (6.5), we can estimate

$$\begin{aligned} \big \Vert D F ({\hat{{\textbf{u}}}}_\varepsilon )\big \Vert ^2_{L^2(B_2)}\overset{(1.9)}{=}\varepsilon ^{-2}\big \Vert f'(|{\textbf{u}}_\varepsilon | )\big \Vert ^2_{L^2(B_{2\varepsilon }(x_1))} \leqslant \varepsilon ^{-2}C_4 \int _{B_{2\varepsilon }(x_1)} F( {\textbf{u}}_\varepsilon )\, dx\overset{(6.2a)}{\leqslant }{\hat{c}}^2 C_4. \nonumber \\ \end{aligned}$$
(6.9)

Altogether, we prove that the terms on the right-hand side of (6.8) is bounded in \(L^2(B_2)\). Invoking the interior estimate for elliptic system (cf. [26, Theorem 4.9]), we obtain

$$\begin{aligned} \Vert {\hat{{\textbf{u}}}}_\varepsilon \Vert _{W^{2,2}(B_1)}\leqslant C_4( {\hat{c}}+\Vert {\hat{{\textbf{u}}}}_\varepsilon \Vert _{L^2(B_2)}). \end{aligned}$$
(6.10)

Note that \(C_4\) is independent of \(\mu \). Now we estimate the last term by

$$\begin{aligned} \Vert {\hat{{\textbf{u}}}}_\varepsilon \Vert _{L^2(B_2)}^2&\leqslant C_4\left( 1+ \varepsilon ^{-2} \int _{B_{2\varepsilon }(x_1) \cap \{ x\mid |{\textbf{u}}_\varepsilon (x)|\geqslant 2 \}} \left( | {\textbf{u}}_\varepsilon |-1\right) ^2\right) \\&\overset{(1.22)}{\leqslant }C_4\left( 1+ \varepsilon ^{-2} \int _{B_{2\varepsilon }(x_1)} f (| {\textbf{u}}_\varepsilon |) \right) \overset{(6.2a)}{\leqslant }(1+{\hat{c}}^2)C_4. \end{aligned}$$

Substituting this estimate in (6.10) and using Morrey’s embedding \(W^{2,2}\hookrightarrow C^{1/2}\), we obtain \(\Vert {\hat{{\textbf{u}}}}_\varepsilon \Vert _{C^{1/2}({\bar{B}}_1)}\leqslant C_4{\hat{c}}\). Rescaling back, we find (6.7) with

$$\begin{aligned} {\hat{C}}:=C_4{\hat{c}}. \end{aligned}$$

Step 2: We claim that with the choice

$$\begin{aligned} {\hat{\epsilon }}< 16^{-8} C_4^{-2}{\hat{c}}^{-2}=16^{-8} {\hat{C}}^{-2}, \end{aligned}$$
(6.11)

we have either (6.6) or

$$\begin{aligned} |{\textbf{u}}_\varepsilon | \leqslant 1/4 \text { on } B_r\quad \text { for } \varepsilon \leqslant r/4. \end{aligned}$$
(6.12)

Indeed, if neither of them were valid, then

$$\begin{aligned} \exists ~\varepsilon \in (0,r/4)\text { and } x_1\in B_r~\text { s.t. }~|{\textbf{u}}_\varepsilon (x_1)|\in (1/4,3/4)\cup (5/4,+\infty ). \end{aligned}$$
(6.13)

Since \(16 {\hat{\epsilon }}<1\), it follows from (6.7) that

$$\begin{aligned} |{\textbf{u}}_\varepsilon (x_1)-{\textbf{u}}_\varepsilon (x)| < 4^{-7}\qquad \text { for } x\in B_{ 16 {\hat{\epsilon }}\varepsilon }(x_1). \end{aligned}$$
(6.14)

Using this and (1.22), we deduce one of the following two cases for \(x\in B_{16 {\hat{\epsilon }}\varepsilon }(x_1)\):

a) If \(|{\textbf{u}}_\varepsilon (x_1)|>3\), then \(|{\textbf{u}}_\varepsilon (x)|>2\) and thus \(f( |{\textbf{u}}_\varepsilon (x)|)\geqslant 1\).

b) If \(|{\textbf{u}}_\varepsilon (x_1)|\in (1/4,3/4)\cup (5/4,3]\), then \(f(|{\textbf{u}}_\varepsilon (x_1)|)\geqslant 1/{16}\). By the third condition in (1.22) and (6.14), we have \(f(|{\textbf{u}}_\varepsilon (x)|)> 1/{32}\).

To summarize, we have the following inequality:

$$\begin{aligned} F({\textbf{u}}_\varepsilon (x))=f(|{\textbf{u}}_\varepsilon (x)|)> 1/{32}\qquad \forall x\in B_{16{\hat{\epsilon }}\varepsilon }(x_1). \end{aligned}$$
(6.15)

Integrating this inequality over \(B_{16{\hat{\epsilon }}\varepsilon }(x_1)\) and using the assumption \(\varepsilon < r/4\), we find

$$\begin{aligned} \varepsilon ^{-2}\int _{B_{ 16{\hat{\epsilon }}\varepsilon }(x_1)} F({\textbf{u}}_\varepsilon (x))\,dx> 8\pi {\hat{\epsilon }}^2. \end{aligned}$$

However, this would contradict (6.3b) since \(B_{ 16{\hat{\epsilon }}\varepsilon }(x_1)\subset B_{2r}(x_0)\). So (6.13) is not valid and the claim is proved.

Step 3: We shall rule out (6.12).

Assuming (6.12), we deduce from (1.22) that \(F({\textbf{u}}_\varepsilon )=|{\textbf{u}}_\varepsilon |^2\). By (6.1) we have

$$\begin{aligned} \mu \nabla ({\text {div}}{\textbf{u}}_\varepsilon ) +\Delta {\textbf{u}}_\varepsilon - 2\varepsilon ^{-2} {\textbf{u}}_\varepsilon =\varvec{\tau }_\varepsilon \,\,\,\text {in}~ B_r. \end{aligned}$$
(6.16)

For \(z\in B_1\), we introduce \({\widetilde{{\textbf{u}}}}_\varepsilon (z):={\textbf{u}}_\varepsilon (rz)\) and \({\widetilde{\varvec{\tau }}}_\varepsilon (z):=\varvec{\tau }_\varepsilon (rz)\). Then

$$\begin{aligned} \mu \nabla ({\text {div}}{\widetilde{{\textbf{u}}}}_\varepsilon ) +\Delta {\widetilde{{\textbf{u}}}}_\varepsilon - 2r^2\varepsilon ^{-2} {\widetilde{{\textbf{u}}}}_\varepsilon =r^2{\widetilde{\varvec{\tau }}}_\varepsilon \,\,\,\text {in}~ B_1. \end{aligned}$$
(6.17)

By the interior estimate for elliptic system, we have

$$\begin{aligned} \Vert {\widetilde{{\textbf{u}}}}_\varepsilon \Vert _{W^{2,2}(B_{1/2 })}+r^2\varepsilon ^{-2}\Vert {\widetilde{{\textbf{u}}}}_\varepsilon \Vert _{L^2(B_{1/2 })}\leqslant C_4\left( \Vert {\widetilde{\varvec{\tau }}}_\varepsilon \Vert _{L^2(B_1)}+\Vert {\widetilde{{\textbf{u}}}}_\varepsilon \Vert _{L^2(B_1)}\right) . \end{aligned}$$
(6.18)

Indeed, one can adapt the proof of [26, Theorem 4.9] to gain the term \(r^2\varepsilon ^{-2}\Vert {\widetilde{{\textbf{u}}}}_\varepsilon \Vert _{L^2(B_{1/2 })}\). By (6.18), (6.2a) and the conclusion in step 2, we find

$$\begin{aligned} r^2\varepsilon ^{-2} \Vert {\textbf{u}}_\varepsilon \Vert _{L^2(B_{r/2})}\leqslant C_4\left( \Vert \varvec{\tau }_\varepsilon \Vert _{L^2(B_r)}+\Vert {\textbf{u}}_\varepsilon \Vert _{L^2(B_r)}\right) \leqslant C_4({\hat{c}}+1). \end{aligned}$$

This implies that \({\textbf{u}}_\varepsilon \rightarrow 0\) strongly in \(L^2(B_{r/2})\), which contradicts (4.2) since \(B_{r/2}\subset K\subset \subset \Omega _t^+\). Therefore, we rule out (6.12) and obtain (6.6). \(\square \)

By (6.6), we have polar decomposition \({\textbf{u}}_\varepsilon =\rho _\varepsilon {\textbf{v}}_\varepsilon \) where

$$\begin{aligned} \rho _\varepsilon =|{\textbf{u}}_\varepsilon |,\quad {\textbf{v}}_\varepsilon ={\textbf{u}}_\varepsilon /|{\textbf{u}}_\varepsilon |~\text { in } ~B_r(x_0). \end{aligned}$$
(6.19)

We set

$$\begin{aligned} {\textbf{w}}_\varepsilon :=({\textbf{v}}_\varepsilon , \rho _\varepsilon ), \end{aligned}$$
(6.20)

and define the projection

$$\begin{aligned} {\textbf{a}}_{\Vert }:=({\mathbb {I}}_3-{\textbf{v}}_\varepsilon \otimes {\textbf{v}}_\varepsilon ){\textbf{a}} \end{aligned}$$
(6.21)

for a vector field \({\textbf{a}}\).

Lemma 6.2

Under the assumptions \(\varepsilon \leqslant r/4\), (6.2a) and (6.3b) for \({\hat{\epsilon }}\) defined in (6.11), \(\rho _\varepsilon \) satisfies the following equation in \(B_r(x_0)\).

$$\begin{aligned} \Delta \rho _\varepsilon -\varepsilon ^{-2} f'(\rho _\varepsilon )&+ \mu \nabla ^2\rho _\varepsilon : ({\textbf{v}}_\varepsilon \otimes {\textbf{v}}_\varepsilon ) +\mu \rho _\varepsilon ({\textbf{v}}_\varepsilon \cdot \nabla ) {\text {div}}{\textbf{v}}_\varepsilon \nonumber \\&=\varvec{\tau }_\varepsilon \cdot {\textbf{v}}_\varepsilon + {\mathcal {N}}_{1,\varepsilon }(\nabla {\textbf{w}}_\varepsilon ,\nabla {\textbf{w}}_\varepsilon ), \end{aligned}$$
(6.22)

where \({\mathcal {N}}_{1,\varepsilon }(\cdot ,\cdot ):{\mathbb {R}}^{4\times 3}\times {\mathbb {R}}^{4\times 3}\mapsto {\mathbb {R}}\) is bilinear with uniformly bounded coefficients. Also, \({\textbf{v}}_\varepsilon \) satisfies the following equation in \(B_r(x_0)\).

$$\begin{aligned} \rho _\varepsilon \Delta {\textbf{v}}_\varepsilon&+\mu ~ ((\nabla ^2\rho _\varepsilon ) {\textbf{v}}_\varepsilon )_{\Vert } +\mu \rho _\varepsilon \left( \nabla ({\text {div}}{\textbf{v}}_\varepsilon )\right) _{\Vert }\nonumber \\&= (\varvec{\tau }_\varepsilon )_{\Vert } + {\mathcal {N}}_{2,\varepsilon }(\nabla {\textbf{w}}_\varepsilon ,\nabla {\textbf{w}}_\varepsilon ), \end{aligned}$$
(6.23)

where \({\mathcal {N}}_{2,\varepsilon }(\cdot ,\cdot ):{\mathbb {R}}^{4\times 3}\times {\mathbb {R}}^{4\times 3}\mapsto {\mathbb {R}}^3\) is bilinear with uniformly bounded coefficients.

Proof

To simplify the presentation we will suppress the subscript \(\varepsilon \). By (6.19) we have \(|{\textbf{v}}|^2\equiv 1\) and thus

$$\begin{aligned} \Delta {\textbf{v}}\cdot {\textbf{v}}=-|\nabla {\textbf{v}}|^2. \end{aligned}$$
(6.24)

Substituting (6.19) into (6.1), we find

$$\begin{aligned} \varvec{\tau }=&~(\Delta \rho ) {\textbf{v}}+2(\nabla \rho \cdot \nabla ) {\textbf{v}}+\rho \Delta {\textbf{v}}-\varepsilon ^{-2} f'(\rho ) {\textbf{v}}\nonumber \\&+\mu ~ (\nabla ^2\rho ) {\textbf{v}}+\mu \rho \nabla ({\text {div}}{\textbf{v}}) +\mu \left( \nabla \rho \cdot \partial _i {\textbf{v}}\right) _{1\leqslant i\leqslant 3} +\mu \nabla \rho ({\text {div}}{\textbf{v}}). \end{aligned}$$
(6.25)

Testing (6.25) with \({\textbf{v}}\) and using (6.24), we obtain

$$\begin{aligned}&- \Delta \rho +\varepsilon ^{-2} f'(\rho )\nonumber \\&\quad = - \varvec{\tau }\cdot {\textbf{v}}+\mu \nabla ^2\rho : ({\textbf{v}}\otimes {\textbf{v}}) +\mu \rho ({\textbf{v}}\cdot \nabla ) {\text {div}}{\textbf{v}}\nonumber \\&\qquad +\mu (\nabla \rho \cdot \partial _i {\textbf{v}})v_i+\mu ({\textbf{v}}\cdot \nabla \rho ){\text {div}}{\textbf{v}}-\rho |\nabla {\textbf{v}}|^2. \end{aligned}$$
(6.26)

The terms in the last line are bilinear with respect to \(\nabla {\textbf{w}}=(\nabla {\textbf{v}},\nabla \rho )\), and we denote their sum by \(-{\mathcal {N}}_{1,\varepsilon }(\nabla {\textbf{w}},\nabla {\textbf{w}})\). By (6.6), it has bounded coefficients and thus (6.22) is proved.

To derive (6.23), we shall use the following identities.

$$\begin{aligned} {\textbf{v}}_{\Vert }=0~\text { and }~ (\partial _i {\textbf{v}})_{\Vert }=\partial _i {\textbf{v}}. \end{aligned}$$
(6.27)

These combined with (6.24) lead to

$$\begin{aligned} (\Delta {\textbf{v}})_{\Vert }=\Delta {\textbf{v}}+|\nabla {\textbf{v}}|^2{\textbf{v}}. \end{aligned}$$
(6.28)

Now applying \((\cdot )_{\Vert }\) to the equation in (6.25), and using (6.27) and (6.28), we obtain

$$\begin{aligned} \varvec{\tau }_{\Vert } =&~ \rho \Delta {\textbf{v}}+\mu ~ ((\nabla ^2\rho ) {\textbf{v}})_{\Vert } +\mu \rho \left( \nabla ({\text {div}}{\textbf{v}})\right) _{\Vert } \nonumber \\&+ 2((\nabla \rho \cdot \nabla ) {\textbf{v}})_{\Vert } +\rho {\textbf{v}}|\nabla {\textbf{v}}|^2+\mu \Big (\left( \nabla \rho \cdot \partial _i {\textbf{v}}\right) _{1\leqslant i\leqslant 3}\Big )_{\Vert } +\mu (\nabla \rho )_{\Vert } ({\text {div}}{\textbf{v}}). \end{aligned}$$
(6.29)

The terms in the second line of the above equation are bilinear with respect to \(\nabla {\textbf{w}}\), and we denote their sum by \(-{\mathcal {N}}_{2,\varepsilon }(\nabla {\textbf{w}},\nabla {\textbf{w}})\). By (6.6), it has bounded coefficients, and thus (6.23) is proved. \(\square \)

Proof of Proposition 6.1

We first show that, by choosing \({\hat{\epsilon }}\) and \(\mu \) sufficiently small, we have

$$\begin{aligned} \Vert \nabla ^2 ({\textbf{v}}_\varepsilon , \rho _\varepsilon )\Vert _{L^{4/3}(B_{r/2}(x_0))} \leqslant {2} C_4r^{-2}. \end{aligned}$$
(6.30)

Recalling (6.20), we deduce from (6.3b) and (6.6) that

$$\begin{aligned} \Vert \nabla {\textbf{w}}_\varepsilon \Vert _{L^2( B_r(x_0))}\leqslant 4{\hat{\epsilon }}~ \text { on } B_r(x_0)~ \text { for } \varepsilon \leqslant r/4. \end{aligned}$$
(6.31)

Recalling that \(r<1\), let \(\chi \) be a \(C^2\) cut-off function such that

$$\begin{aligned} \chi \equiv {\left\{ \begin{array}{ll} 1\text { in }B_{r/2}(x_0)\\ 0\text { in } {B_1(x_0)}\backslash B_r(x_0) \end{array}\right. } \text { and } |\nabla ^\ell \chi |\leqslant 8r^{-\ell } \text { in } B_1(x_0)\text { for } \ell \in \{1,2\}. \end{aligned}$$
(6.32)

and let \({\bar{{\textbf{w}}}}_\varepsilon :=({\bar{{\textbf{v}}}}_\varepsilon ,{\bar{\rho }}_\varepsilon )\) with

$$\begin{aligned} {\bar{\rho }}_\varepsilon =\chi (\rho _\varepsilon -1)\quad \text { and }\quad {\bar{{\textbf{v}}}}_\varepsilon =\chi {\textbf{v}}_\varepsilon . \end{aligned}$$
(6.33)

Multiplying (6.23) by \(\chi \) and using the linearity of \({\textbf{a}}_{\Vert }\) about \({\textbf{a}}\) (cf. (6.21)), we find

$$\begin{aligned} \rho _\varepsilon \Delta {\bar{{\textbf{v}}}}_\varepsilon +&\rho _\varepsilon [\chi ,\Delta ] {\textbf{v}}_\varepsilon + \mu \Big ([\chi ,\nabla ^2] (\rho _\varepsilon -1) {\textbf{v}}_\varepsilon \Big )_{\Vert }+ \mu \Big ( \nabla ^2 {\bar{\rho }}_\varepsilon {\textbf{v}}_\varepsilon \Big )_{\Vert } \nonumber \\&+\mu \rho _\varepsilon ~ \Big ([\chi ,\nabla {\text {div}}] {\textbf{v}}_\varepsilon \Big )_{\Vert } +\mu \rho _\varepsilon ~\Big ( \nabla ({\text {div}}{\bar{{\textbf{v}}}}_\varepsilon )\Big )_{\Vert }\nonumber \\&= (\varvec{\tau }_\varepsilon )_{\Vert } \chi + {\mathcal {N}}_{2,\varepsilon }(\chi \nabla {\textbf{w}}_\varepsilon ,\nabla {\textbf{w}}_\varepsilon )\qquad \text { in } B_1(x_0), \end{aligned}$$
(6.34)

and \({\bar{{\textbf{v}}}}_\varepsilon |_{\partial B_1(x_0)}=0\). For brevity we denote \(L^p(B_1(x_0))\) by \(L^p\). Note that the commutators in (6.34) involve at most first order derivatives of \({\textbf{w}}_\varepsilon =({\textbf{v}}_\varepsilon ,\rho _\varepsilon )\), which satisfies (6.31). Now applying the \(L^p\)-estimate for elliptic equation [39, pp. 109] (componentwise) in (6.34), and invoking (6.31) and (6.6), we have

$$\begin{aligned} \Vert \nabla ^2 {\bar{{\textbf{v}}}}_\varepsilon \Vert _{L^{4/3}} \leqslant ~&C_4\Big ( r^{-2} +r^{-1}+ \mu \Vert \nabla ^2 {\bar{{\textbf{w}}}}_\varepsilon \Vert _{L^{4/3}}+\left\| {\mathcal {N}}_{2,\varepsilon }(\chi \nabla {\textbf{w}}_\varepsilon ,\nabla {\textbf{w}}_\varepsilon )\right\| _{L^{4/3}}\Big ). \end{aligned}$$
(6.35)

Note that the prefactors \(r^{-1}\) and \(r^{-2}\) are due to the differentiation of \(\chi \) (cf. (6.32)), and that \(C_4\) is independent of r. To estimate the last term, we employ the bi-linearity of \({\mathcal {N}}_{2,\varepsilon }\), (6.31) and (6.6):

$$\begin{aligned} \begin{aligned}&\left\| {\mathcal {N}}_{2,\varepsilon }(\chi \nabla {\textbf{w}}_\varepsilon ,\nabla {\textbf{w}}_\varepsilon )\right\| _{L^{4/3}}\\&\quad \leqslant \left\| {\mathcal {N}}_{2,\varepsilon }( \nabla {\bar{{\textbf{w}}}}_\varepsilon ,\nabla {\textbf{w}}_\varepsilon )\right\| _{L^{4/3}}+\left\| {\mathcal {N}}_{2,\varepsilon }( \nabla \chi \otimes {\textbf{w}}_\varepsilon ,\nabla {\textbf{w}}_\varepsilon )\right\| _{L^{4/3}}+C_4 r^{-1}\\&\quad \leqslant C_4\left( \Vert \nabla {\bar{{\textbf{w}}}}_\varepsilon \Vert _{L^4}\Vert \nabla {\textbf{w}}_\varepsilon \Vert _{L^2}+r^{-1}\right) \\&\quad \leqslant C_4\left( \Vert \nabla ^2{\bar{{\textbf{w}}}}_\varepsilon \Vert _{L^{4/3}}\Vert \nabla {\textbf{w}}_\varepsilon \Vert _{L^2}+r^{-1}\right) . \end{aligned} \end{aligned}$$
(6.36)

Note that in the last step we used the Sobolev’s embedding \(W^{1,4/3}(B_1)\subset L^4(B_1)\). Combining (6.36) with (6.35), we obtain

$$\begin{aligned} \Vert \nabla ^2 {\bar{{\textbf{v}}}}_\varepsilon \Vert _{L^{4/3}} \leqslant ~&C_4\left( r^{-2}+ \mu \Vert \nabla ^2 ({\bar{{\textbf{v}}}}_\varepsilon ,{\bar{\rho }}_\varepsilon )\Vert _{L^{4/3}}+ { \Vert \nabla ^2{\bar{{\textbf{w}}}}_\varepsilon \Vert _{L^{4/3}}}\Vert \nabla {\textbf{w}}_\varepsilon \Vert _{L^2}\right) . \end{aligned}$$
(6.37)

Now we turn to the estimate of \(\rho _\varepsilon \). Using (6.6) and (1.22), we have \(f'(\rho _\varepsilon )=2(\rho _\varepsilon -1)\) in \(B_r(x_0)\). Now multiplying (6.22) by \(\chi \) and using the linearity of (6.21), we find

$$\begin{aligned}&-2\varepsilon ^{-2} {\bar{\rho }}_\varepsilon +\Delta {\bar{\rho }}_\varepsilon +[\chi ,\Delta ] (\rho _\varepsilon -1) +\mu ({\textbf{v}}_\varepsilon \otimes {\textbf{v}}_\varepsilon ): \nabla ^2{\bar{\rho }}_\varepsilon \\&\quad \quad +\mu ({\textbf{v}}_\varepsilon \otimes {\textbf{v}}_\varepsilon ): [\chi ,\nabla ^2](\rho _\varepsilon -1) +\mu \rho _\varepsilon {\textbf{v}}_\varepsilon \cdot (\nabla {\text {div}}{\bar{{\textbf{v}}}}_\varepsilon ) +\mu \rho _\varepsilon {\textbf{v}}_\varepsilon \cdot \left( [\chi ,\nabla {\text {div}}]{\textbf{v}}_\varepsilon \right) \\&\quad =\chi \varvec{\tau }_\varepsilon \cdot {\textbf{v}}_\varepsilon + {\mathcal {N}}_{1,\varepsilon }(\chi \nabla {\textbf{w}}_\varepsilon ,\nabla {\textbf{w}}_\varepsilon ). \end{aligned}$$

In the same way as we did for (6.37), we find

$$\begin{aligned} \Vert \nabla ^2 {\bar{\rho }}_\varepsilon \Vert _{L^{4/3}} \leqslant ~&C_4\left( r^{-2}+ \mu \Vert \nabla ^2 ({\bar{{\textbf{v}}}}_\varepsilon ,{\bar{\rho }}_\varepsilon )\Vert _{L^{4/3}}+\left\| {\mathcal {N}}_{1,\varepsilon }(\chi \nabla {\textbf{w}}_\varepsilon ,\nabla {\textbf{w}}_\varepsilon )\right\| _{L^{4/3}}\right) \\ \leqslant ~&C_4\left( r^{-2}+\mu \Vert \nabla ^2 ({\bar{{\textbf{v}}}}_\varepsilon ,{\bar{\rho }}_\varepsilon )\Vert _{L^{4/3}}+ \Vert \nabla ^2{\bar{{\textbf{w}}}}_\varepsilon \Vert _{L^{4/3}}\Vert \nabla {\textbf{w}}_\varepsilon \Vert _{L^2}\right) . \end{aligned}$$

Combining this with (6.37) and (6.31) we discover

$$\begin{aligned} \Vert \nabla ^2 ({\bar{{\textbf{v}}}}_\varepsilon ,{\bar{\rho }}_\varepsilon )\Vert _{L^{4/3}(B_r(x_0))} \leqslant C_4\left( r^{-2}+ \max \{ {\hat{\epsilon }}, \mu \}\Vert \nabla ^2({\bar{{\textbf{v}}}}_\varepsilon ,{\bar{\rho }}_\varepsilon )\Vert _{L^{4/3}(B_r(x_0))}\right) . \end{aligned}$$
(6.38)

Note that before Lemma 6.1, we have assumed that \(C_4>1\) and \({\hat{c}}>1\). Recall also the choice of \({\hat{\epsilon }}\) in (6.11). By choosing

$$\begin{aligned} \Lambda = 16^{-8} C_4^{-2}\text { in} (6.3a), \end{aligned}$$

we find \(C_4 \max \{ {\hat{\epsilon }}, \mu \}<1/2\). This combined with (6.38) yields

$$\begin{aligned} \Vert \nabla ^2 ({\bar{{\textbf{v}}}}_\varepsilon ,{\bar{\rho }}_\varepsilon )\Vert _{L^{4/3}(B_r(x_0))}\leqslant 2C_4 r^{-2}. \end{aligned}$$

In view of (6.32) and (6.33), this implies (6.30).

Now using (6.2b), we have \(\rho _{\varepsilon _k}(\cdot ,t_0)\xrightarrow {k\rightarrow \infty } |{\textbf{u}}|(\cdot ,t_0)=1\) strongly in \(L^2(B_r(x_0))\). Thus, using (6.6) we find

$$\begin{aligned} \Vert {\textbf{v}}_{\varepsilon _k}-{\textbf{u}}\Vert ^2_{L^2(B_r(x_0))}\leqslant 2\Vert {\textbf{u}}_{\varepsilon _k}-{\textbf{u}}\rho _{\varepsilon _k}\Vert ^2_{L^2(B_r(x_0))}\xrightarrow {k\rightarrow \infty } 0. \end{aligned}$$

These together with (6.30) and the Gagliardo-Nirenberg interpolation inequality yield

$$\begin{aligned} \left( {\textbf{v}}_{\varepsilon _k}, \rho _{\varepsilon _k} \right) \xrightarrow {k\rightarrow \infty } \left( {\textbf{u}}, 1\right) \text { strongly in } W^{1,2}(B_{r/2}(x_0)). \end{aligned}$$
(6.39)

Finally, using (6.6) and (6.39) we find

$$\begin{aligned} \nabla {\textbf{u}}_{\varepsilon _k}= \rho _{\varepsilon _k} \nabla {\textbf{v}}_{\varepsilon _k}+{\textbf{v}}_{\varepsilon _k} \nabla \rho _{\varepsilon _k} \xrightarrow {k\rightarrow \infty } \nabla {\textbf{u}}\text { strongly in }L^2(B_{r/2}(x_0)), \end{aligned}$$

and finish the proof of (6.4). \(\square \)

Proof of Theorem 1.2

We employ the covering argument in [14]. For any test function \(\varvec{\Psi }\in C_c^1(\Omega _t^+;{\mathbb {R}}^3)\), we choose \(K=\overline{\textrm{supp} (\varvec{\Psi })}\subset \subset \Omega _t^+\), and we define the singular set at time \(t \in (0, T]\) by

$$\begin{aligned} \Sigma _t:=\bigcap _{0<r<1}\left\{ x \in K\mid B_{2r}(x)\subset K, \varliminf _{k \rightarrow \infty } \int _{B_{2r} (x )}\left( \frac{1}{2}\left| \nabla {\textbf{u}}_{\varepsilon _k}\right| ^{2}+\frac{F( {\textbf{u}}_{\varepsilon _k})}{\varepsilon _{k}^2} \right) \, dx>\frac{{\hat{\epsilon }}^2}{2}\right\} . \end{aligned}$$
(6.40)

We claim that \(\Sigma _t\) is discrete. Indeed, choose an arbitrary finite set \(\{y_j\}_{j=1}^J\subset \Sigma _t\) with mutually disjoint balls \(\{B_{2r_j}(y_j)\}_{j=1}^J\) inside K with \(r_j<1/2\). Since J is finite, there exists \(k_J>0\) such that for any \(k\geqslant k_J\) we have

$$\begin{aligned} \int _{B_{2r_j} (y_j )}\left( \frac{1}{2}\left| \nabla {\textbf{u}}_{\varepsilon _k}\right| ^{2}+\frac{F( {\textbf{u}}_{\varepsilon _k})}{\varepsilon _{k}^2} \right) \, dx>\frac{{\hat{\epsilon }}^2}{4}\quad \text { for all }j\in \{1,\cdots , J\}. \end{aligned}$$
(6.41)

Combined with (6.2a), this implies

$$\begin{aligned} {\hat{c}}^2\geqslant \int _{\bigsqcup _{j=1}^J B_{2r_j} (y_j )}\left( \frac{1}{2}\left| \nabla {\textbf{u}}_{\varepsilon _k}\right| ^{2}+\frac{F( {\textbf{u}}_{\varepsilon _k})}{\varepsilon _{k}^2} \right) \, dx>\frac{{\hat{\epsilon }}^2}{4}J. \end{aligned}$$
(6.42)

As a result, \(J\leqslant 4{\hat{c}}^2{\hat{\epsilon }}^{-2}\) and thus \(\Sigma _t\) is discrete. Therefore w.l.o.g. we can assume that \(\Sigma _t=\{x_0\}\) and \(B_{2r}(x_0)\subset K\). Let \(\eta \in C_c^1(B_2(0))\) be a cut-off function which \(\equiv 1\) in \(B_1(0)\). Then

$$\begin{aligned} \varvec{\Psi }_\delta (x):=\varvec{\Psi }(x)\left( 1-\eta (\tfrac{ x-x_0}{\delta })\right) \xrightarrow {\delta \rightarrow 0} \varvec{\Psi }(x)\text { for any }x\ne x_0. \end{aligned}$$
(6.43)

It is obvious that \(\varvec{\Psi }_\delta =0\) in \(B_{\delta }(x_0)\). By (6.40) and Proposition 6.1, we have

$$\begin{aligned} \nabla {\textbf{u}}_{\varepsilon _k}\xrightarrow {k\rightarrow \infty } \nabla {\textbf{u}}\text { strongly in }L^2(K\backslash B_\delta (x_0)). \end{aligned}$$
(6.44)

Using these properties, we can apply \( \wedge {\textbf{u}}_{\varepsilon _k} \cdot \varvec{\Psi }_\delta \) to both sides of (1.2a), integrate by parts and then send \(k\rightarrow \infty \):

$$\begin{aligned}&\int _\Omega \partial _t {\textbf{u}}\wedge {\textbf{u}}\cdot \varvec{\Psi }_\delta \,dx+\mu \int _\Omega \left( {\text {div}}{\textbf{u}}\right) ({\text {rot}}{\textbf{u}}) \cdot \varvec{\Psi }_\delta \,dx\nonumber \\&+\int _\Omega (\nabla {\textbf{u}}\wedge {\textbf{u}})\cdot \nabla \varvec{\Psi }_\delta \,dx-\mu \int _\Omega \left( {\text {div}}{\textbf{u}}\right) ( {\text {rot}}\varvec{\Psi }_\delta )\cdot {\textbf{u}}\,dx=0. \end{aligned}$$
(6.45)

Note that we have also used \(\partial _t {\textbf{u}}_{\varepsilon _k} \wedge {\textbf{u}}_{\varepsilon _k}\xrightarrow {k\rightarrow \infty } \partial _t {\textbf{u}}\wedge {\textbf{u}}\) weakly in \(L^2(0,T;L^{6/5}(\Omega ))\), which is due to Proposition 4.1. By (6.43) and the regularity of \({\textbf{u}}\) (cf. (4.26a) and (4.26b)), we can send \(\delta \rightarrow 0\) in the first and the second integrals in (6.45) using the dominated convergence theorem. Concerning the third one, we have

$$\begin{aligned} \int _\Omega (\nabla {\textbf{u}}\wedge {\textbf{u}})\cdot \nabla \varvec{\Psi }_\delta \,dx&= \int _\Omega \left( 1-\eta (\tfrac{ x-x_0}{\delta })\right) (\nabla {\textbf{u}}\wedge {\textbf{u}})\cdot \nabla \varvec{\Psi }\,dx\nonumber \\&\quad - \sum _{i=1}^3\int _{B_{2\delta }(x_0)} \frac{1}{\delta }(\partial _i\eta )(\tfrac{ x-x_0}{\delta }) ~\partial _i {\textbf{u}}\wedge {\textbf{u}}\cdot \varvec{\Psi }\,dx.\nonumber \\ \end{aligned}$$
(6.46)

We claim that the second integral on the right-hand side vanishes as \(\delta \rightarrow 0\). Indeed, by the Cauchy–Schwarz inequality we have

$$\begin{aligned}&\left| \sum _{i=1}^3\int _{B_{2\delta }(x_0)} \frac{1}{\delta }(\partial _i\eta )(\tfrac{ x-x_0}{\delta }) ~\partial _i {\textbf{u}}\wedge {\textbf{u}}\cdot \varvec{\Psi }\,dx\right| \nonumber \\&\quad \leqslant C\Vert \varvec{\Psi }\Vert _{L^\infty } \Vert \nabla \eta \Vert _{L^2(B_2)}\Vert \nabla {\textbf{u}}\Vert _{L^2(B_{2\delta }(x_0))}\xrightarrow {\delta \rightarrow 0}0. \end{aligned}$$
(6.47)

Now using \(\lim _{\delta \rightarrow 0}\eta (\tfrac{ x-x_0}{\delta })= 0\) for any \(x\ne x_0\), we can send \(\delta \rightarrow 0\) in (6.46) and obtain

$$\begin{aligned} \int _\Omega (\nabla {\textbf{u}}\wedge {\textbf{u}})\cdot \nabla \varvec{\Psi }_\delta \,dx\xrightarrow {\delta \rightarrow 0}\int _\Omega (\nabla {\textbf{u}}\wedge {\textbf{u}})\cdot \nabla \varvec{\Psi }\,dx. \end{aligned}$$

By the same argument we can compute the fourth integral in (6.45) and find

$$\begin{aligned} \int _\Omega \left( {\text {div}}{\textbf{u}}\right) ( {\text {rot}}\varvec{\Psi }_\delta )\cdot {\textbf{u}}\,dx \xrightarrow {\delta \rightarrow 0}\int _\Omega \left( {\text {div}}{\textbf{u}}\right) ( {\text {rot}}\varvec{\Psi })\cdot {\textbf{u}}\,dx. \end{aligned}$$
(6.48)

Using the above two formulas, we can send \(\delta \rightarrow 0\) in (6.45) and obtain (1.23). \(\square \)